9694 thinking skills - maxpapersmaxpapers.com/wp-content/uploads/2012/11/9694_w14_ms_complete… ·...

71
® IGCSE is the registered trademark of Cambridge International Examinations. CAMBRIDGE INTERNATIONAL EXAMINATIONS Cambridge International Advanced Subsidiary and Advanced Level MARK SCHEME for the October/November 2014 series 9694 THINKING SKILLS 9694/11 Paper 1 (Problem Solving), maximum raw mark 30 Mark schemes should be read in conjunction with the question paper and the Principal Examiner Report for Teachers. Cambridge will not enter into discussions about these mark schemes. Cambridge is publishing the mark schemes for the October/November 2014 series for most Cambridge IGCSE ® , Cambridge International A and AS Level components and some Cambridge O Level components.

Upload: ngotuong

Post on 26-Apr-2018

311 views

Category:

Documents


5 download

TRANSCRIPT

Page 1: 9694 THINKING SKILLS - maxpapersmaxpapers.com/wp-content/uploads/2012/11/9694_w14_ms_complete… · 9694 THINKING SKILLS 9694/12 Paper 1 ... October/November 2014 9694 12 ... 1-mark

® IGCSE is the registered trademark of Cambridge International Examinations.

CAMBRIDGE INTERNATIONAL EXAMINATIONS

Cambridge International Advanced Subsidiary and Advanced Level

MARK SCHEME for the October/November 2014 series

9694 THINKING SKILLS

9694/11 Paper 1 (Problem Solving), maximum raw mark 30

Mark schemes should be read in conjunction with the question paper and the Principal Examiner Report for Teachers. Cambridge will not enter into discussions about these mark schemes. Cambridge is publishing the mark schemes for the October/November 2014 series for most Cambridge IGCSE

®, Cambridge International A and AS Level components and some

Cambridge O Level components.

Page 2: 9694 THINKING SKILLS - maxpapersmaxpapers.com/wp-content/uploads/2012/11/9694_w14_ms_complete… · 9694 THINKING SKILLS 9694/12 Paper 1 ... October/November 2014 9694 12 ... 1-mark

Page 2 Mark Scheme Syllabus Paper

Cambridge International AS/A Level – October/November 2014 9694 11

© Cambridge International Examinations 2014

Question Number

Key Question Number

Key

1 A 16 C

2 C 17 C

3 C 18 B

4 B 19 D

5 C 20 B

6 C 21 D

7 C 22 C

8 A 23 C

9 D 24 D

10 B 25 D

11 B 26 C

12 D 27 C

13 B 28 B

14 B 29 C

15 C 30 C

Page 3: 9694 THINKING SKILLS - maxpapersmaxpapers.com/wp-content/uploads/2012/11/9694_w14_ms_complete… · 9694 THINKING SKILLS 9694/12 Paper 1 ... October/November 2014 9694 12 ... 1-mark

© Cambridge International Examinations 2016

CAMBRIDGE INTERNATIONAL EXAMINATIONS

Cambridge International Advanced Subsidiary and Advanced Level

MARK SCHEME UPDATE NOTICE

for the October/November 2014 series

9694 THINKING SKILLS

9694/11 Paper 1 (Problem Solving)

Please note that there was a typographical error in the published mark scheme for 9694/11 for the November 2014 series: the answer to Question 9 should have been D, not A. A corrected version was uploaded on 27th January 2016. Please accept our apologies for any inconvenience, and be assured that the candidates’ work was marked correctly.

Page 4: 9694 THINKING SKILLS - maxpapersmaxpapers.com/wp-content/uploads/2012/11/9694_w14_ms_complete… · 9694 THINKING SKILLS 9694/12 Paper 1 ... October/November 2014 9694 12 ... 1-mark

® IGCSE is the registered trademark of Cambridge International Examinations.

CAMBRIDGE INTERNATIONAL EXAMINATIONS

Cambridge International Advanced Subsidiary and Advanced Level

MARK SCHEME for the October/November 2014 series

9694 THINKING SKILLS

9694/12 Paper 1 (Problem Solving), maximum raw mark 30

Mark schemes should be read in conjunction with the question paper and the Principal Examiner Report for Teachers. Cambridge will not enter into discussions about these mark schemes. Cambridge is publishing the mark schemes for the October/November 2014 series for most Cambridge IGCSE

®, Cambridge International A and AS Level components and some

Cambridge O Level components.

Page 5: 9694 THINKING SKILLS - maxpapersmaxpapers.com/wp-content/uploads/2012/11/9694_w14_ms_complete… · 9694 THINKING SKILLS 9694/12 Paper 1 ... October/November 2014 9694 12 ... 1-mark

Page 2 Mark Scheme Syllabus Paper

Cambridge International AS/A Level – October/November 2014 9694 12

© Cambridge International Examinations 2014

Question Number

Key Question Number

Key

1 B 16 B

2 B 17 C

3 D 18 C

4 D 19 C

5 C 20 D

6 A 21 C

7 C 22 B

8 C 23 C

9 C 24 C

10 B 25 C

11 D 26 D

12 C 27 D

13 D 28 B

14 D 29 C

15 B 30 D

Page 6: 9694 THINKING SKILLS - maxpapersmaxpapers.com/wp-content/uploads/2012/11/9694_w14_ms_complete… · 9694 THINKING SKILLS 9694/12 Paper 1 ... October/November 2014 9694 12 ... 1-mark

© Cambridge International Examinations 2016

CAMBRIDGE INTERNATIONAL EXAMINATIONS

Cambridge International Advanced Subsidiary and Advanced Level

MARK SCHEME UPDATE NOTICE

for the October/November 2014 series

9694 THINKING SKILLS

9694/12 Paper 1 (Problem Solving)

Please note that there was a typographical error in the published mark scheme for 9694/12 for the November 2014 series: the answer to Question 6 should have been A, not D. A corrected version was uploaded on 27th January 2016. Please accept our apologies for any inconvenience, and be assured that the candidates’ work was marked correctly.

Page 7: 9694 THINKING SKILLS - maxpapersmaxpapers.com/wp-content/uploads/2012/11/9694_w14_ms_complete… · 9694 THINKING SKILLS 9694/12 Paper 1 ... October/November 2014 9694 12 ... 1-mark

® IGCSE is the registered trademark of Cambridge International Examinations.

CAMBRIDGE INTERNATIONAL EXAMINATIONS

Cambridge International Advanced Subsidiary and Advanced Level

MARK SCHEME for the October/November 2014 series

9694 THINKING SKILLS

9694/13 Paper 1 (Problem Solving), maximum raw mark 30

Mark schemes should be read in conjunction with the question paper and the Principal Examiner Report for Teachers. Cambridge will not enter into discussions about these mark schemes. Cambridge is publishing the mark schemes for the October/November 2014 series for most Cambridge IGCSE

®, Cambridge International A and AS Level components and some

Cambridge O Level components.

Page 8: 9694 THINKING SKILLS - maxpapersmaxpapers.com/wp-content/uploads/2012/11/9694_w14_ms_complete… · 9694 THINKING SKILLS 9694/12 Paper 1 ... October/November 2014 9694 12 ... 1-mark

Page 2 Mark Scheme Syllabus Paper

Cambridge International AS/A Level – October/November 2014 9694 13

© Cambridge International Examinations 2014

Question Number

Key Question Number

Key

1 D 16 C

2 C 17 A

3 C 18 B

4 C 19 C

5 D 20 C

6 C 21 D

7 C 22 B

8 A 23 C

9 C 24 C

10 C 25 B

11 D 26 A

12 D 27 A

13 C 28 C

14 B 29 B

15 B 30 C

Page 9: 9694 THINKING SKILLS - maxpapersmaxpapers.com/wp-content/uploads/2012/11/9694_w14_ms_complete… · 9694 THINKING SKILLS 9694/12 Paper 1 ... October/November 2014 9694 12 ... 1-mark

® IGCSE is the registered trademark of Cambridge International Examinations.

CAMBRIDGE INTERNATIONAL EXAMINATIONS

Cambridge International Advanced Subsidiary and Advanced Level

MARK SCHEME for the October/November 2014 series

9694 THINKING SKILLS

9694/21 Paper 2 (Critical Thinking), maximum raw mark 45

This mark scheme is published as an aid to teachers and candidates, to indicate the requirements of the examination. It shows the basis on which Examiners were instructed to award marks. It does not indicate the details of the discussions that took place at an Examiners’ meeting before marking began, which would have considered the acceptability of alternative answers. Mark schemes should be read in conjunction with the question paper and the Principal Examiner Report for Teachers. Cambridge will not enter into discussions about these mark schemes. Cambridge is publishing the mark schemes for the October/November 2014 series for most Cambridge IGCSE

®, Cambridge International A and AS Level components and some

Cambridge O Level components.

Page 10: 9694 THINKING SKILLS - maxpapersmaxpapers.com/wp-content/uploads/2012/11/9694_w14_ms_complete… · 9694 THINKING SKILLS 9694/12 Paper 1 ... October/November 2014 9694 12 ... 1-mark

Page 2 Mark Scheme Syllabus Paper

Cambridge International AS/A Level – October/November 2014 9694 21

© Cambridge International Examinations 2014

1 (a) How useful is Source B in deciding whether Mr Perez was injured deliberately or by accident? Justify your answer. [3]

Of little or no use [1]. There is no suggestion that the game would not be played fairly: the

question is whether two players intended to injure a teacher [1]. Even if two players had

conspired to injure Mr Perez, the team captain would not necessarily have known [1]. Even if

the team captain had known of a conspiracy to injure Mr Perez, he probably would not have

admitted it [1].

(b) How significant would it be if the two boys who injured Mr Perez were not the ones mentioned in Source A? [2]

It would make it less likely that he was injured deliberately [1], but would not make it unlikely

[1].

(c) How reliable is the Deputy Headteacher’s claim in Source C that two players in the school team had deliberately injured Mr Perez? [4]

He has a vested interest to prove to the Headteacher that he was right in his original

allegation/bias against the boys/predisposition to interpret events in the light of what he had

been told [1], and he admits that he did not see exactly what happened [1]. Since he is not a

PE specialist, he probably lacks the expertise to judge on the matter [1], but not necessarily

[1].

However, he was present and therefore had fairly good ability to see [1]. The fact that he

distinguishes carefully between what he did and did not see adds to his reliability [1]. His

admission that Mr Perez did not agree with him increases his reliability [1]. The allegation is

plausible in the light of the Deputy Head’s warning in Source A and Mr Perez’s unpopularity

with senior students [1].

No mark for judgment. Maximum 3 marks if only one side covered.

Page 11: 9694 THINKING SKILLS - maxpapersmaxpapers.com/wp-content/uploads/2012/11/9694_w14_ms_complete… · 9694 THINKING SKILLS 9694/12 Paper 1 ... October/November 2014 9694 12 ... 1-mark

Page 3 Mark Scheme Syllabus Paper

Cambridge International AS/A Level – October/November 2014 9694 21

© Cambridge International Examinations 2014

(d) How do you think Mr Perez was injured? Write a short, reasoned argument to support your conclusion, with critical reference to the evidence provided and considering a plausible alternative conclusion. [6]

Level 3 5–6 marks

A strong answer, which provides a reasoned argument including thorough

evaluation of all or most of the evidence to support an acceptable conclusion

in terms of probability and evaluates the plausibility of at least one alternative

conclusion.

Level 2 3–4 marks

An answer which evaluates some of the evidence, draws an acceptable

conclusion in terms of probability and may mention the plausibility of at least

one alternative conclusion.

Level 1 1–2 marks

A weak answer, which refers to some of the evidence, possibly including a

simple evaluative comment. The conclusion may be unstated or over-stated.

Level 0 0 marks

No credit-worthy material.

Indicative Content

• It is plausible that the two boys did injure Mr Perez deliberately.

• It could have been an unintended consequence of participating in a rough game

involving physical contact,

• or conceivably somewhere between these two extremes (i.e., playing more violently

than usual towards a particular opponent, but not a premeditated attack).

That the injury was deliberate is especially likely if they were also the two boys who

according to Source A had allegedly told fellow-students previously that they were planning

to injure him. However, there is not enough evidence to prove that the injury was not an

accident.

Page 12: 9694 THINKING SKILLS - maxpapersmaxpapers.com/wp-content/uploads/2012/11/9694_w14_ms_complete… · 9694 THINKING SKILLS 9694/12 Paper 1 ... October/November 2014 9694 12 ... 1-mark

Page 4 Mark Scheme Syllabus Paper

Cambridge International AS/A Level – October/November 2014 9694 21

© Cambridge International Examinations 2014

2 (a) The professor in Source A claims that sleeping too little causes health problems. Suggest two alternative explanations for the evidence given in Source A. [4]

2 marks for each clear, valid answer. 1 mark for each vague, incomplete or marginal answer. 2 answers required.

2-mark answers Some health problems compromise the patient’s ability to sleep.

The stressful lifestyle described by the professor may cause both the sleeplessness and the

health problems.

1-mark answers The sleeplessness and the health problems may be coincidental.

Correlation does not necessarily imply causation.

The post hoc fallacy.

(b) Is Source C an argument? Briefly justify your answer. [2]

2 marks for a correct answer with accurate explanation. 1 mark for a correct answer with vague, incomplete or generic explanation. 0 marks for correct answer without explanation. 0 marks for incorrect answer with or without explanation.

2-mark answer Source C is not an argument. It reports evidence resulting from research, but does not draw

a persuasive conclusion from it.

1-mark answers Source C is not an argument, because it does not include a persuasive conclusion.

Source C is not an argument. It reports evidence resulting from research.

(c) How effectively does the comment from Dr Neil Stanley in Source D challenge the claims in Source C? Justify your answer. [3]

Not at all effectively [1]. The fact that many people who sleep for less time than average are

obese is quite compatible with the fact that some of them are of normal size – or even

smaller than average [1]. Source C does not deny that “Not everybody needs a lot of sleep”

[1]. The number of examples quoted is too small to challenge the claims effectively [1]. It is

not clear whether the example of Winston Churchill supports or challenges the claim that

people who sleep for less time than average are more likely to be obese [1].

Page 13: 9694 THINKING SKILLS - maxpapersmaxpapers.com/wp-content/uploads/2012/11/9694_w14_ms_complete… · 9694 THINKING SKILLS 9694/12 Paper 1 ... October/November 2014 9694 12 ... 1-mark

Page 5 Mark Scheme Syllabus Paper

Cambridge International AS/A Level – October/November 2014 9694 21

© Cambridge International Examinations 2014

(d) ‘We should all sleep more, in order to live longer.’ To what extent do you agree with this claim? Write a short, reasoned argument to

support your conclusion, using and evaluating the information provided in Sources A–D. [6]

Level 3 5–6 marks

A reasoned argument, which uses and evaluates all or most of the evidence

provided.

Level 2 3–4 marks

A simple argument, which uses and/or evaluates evidence.

Level 1 1–2 marks

A weak answer, which makes some correct reference to evidence but

consists of opinion and/or assertion rather than argument

or a weak argument which makes no reference to evidence.

Level 0 0 marks

No credit-worthy material.

Indicative content

• Source A does claim that some people would have a longer life-expectancy if they

slept more,

• and Source B supports the claim that people who sleep significantly less than 7

hours have reduced life-expectancy,

• although it is far from clear how the relative risks were calculated

• and the results show that it makes little difference whether you sleep for 6, 7 or 8

hours.

• Sources A and/or B also show that sleeping more than the average is associated

with poor health

• (although not necessarily as cause and effect).

• Source C gives a possible explanation to support this link between low amounts of

sleep and increased risk of death,

• on the assumption that obesity is linked to reduced life expectancy.

• But Source D shows that this does not apply to everyone.

3 (a) Using the exact words from the passage as far as possible, identify the main

conclusion. [2] 2 marks: (but in fact) organisations of all kinds should prioritise public relations and

advertising.

1 mark: Some people think that employing people to work in public relations or advertising is

a waste of money, but in fact organisations of all kinds should prioritise public relations and

advertising.

Page 14: 9694 THINKING SKILLS - maxpapersmaxpapers.com/wp-content/uploads/2012/11/9694_w14_ms_complete… · 9694 THINKING SKILLS 9694/12 Paper 1 ... October/November 2014 9694 12 ... 1-mark

Page 6 Mark Scheme Syllabus Paper

Cambridge International AS/A Level – October/November 2014 9694 21

© Cambridge International Examinations 2014

(b) Using the exact words from the passage as far as possible, identify three reasons used to support the main conclusion. [3]

• The most effective investment which manufacturers can make is to advertise more.

• They should just spend money on advertising.

• It [publicity] is (therefore) the first concern of all politicians.

• (So) police forces should reduce the number of police officers and concentrate their

efforts on public relations.

(c) Evaluate the strength of the reasoning in the argument. In your answer you should

consider any flaws, unstated assumptions and other weaknesses. [5] Marks for each evaluative point as follows, up to a maximum of 5 marks: 2 marks Valid evaluative point, clearly expressed. 1 mark Weak attempt at a valid evaluative point.

Paragraph 1

• The conclusion is a generalisation. The sub-arguments are actually examples.

However, it could be argued that these examples are comprehensive enough to

support the generalising conclusion quite well.

Paragraph 2

• Assumption: that the principal task of manufacturing is to make money rather than to

produce goods which meet a need.

• Assumption: that the product is already satisfactory.

• Restricting the options: there are other possible ways of investing money, apart from

advertising and improving the product (e.g. lowering the price).

Paragraph 3

• Assumption: that potential clients are more likely to hear from advertisers than

personal recommendation.

Paragraph 4

• Begins with an ad hominem attack on “impractical idealists”.

• Confuses the means of politics (winning an election) with the end.

Paragraph 5

• The argument depends crucially on the claim “public safety is a state of mind”, but

this claim is controversial and extreme.

• Allow (1 mark only) Assumption: that conviction rates will be high and crime rates

will be low.

• Assumption: that benefits gained by employing one public relations officer can be

extrapolated to larger numbers.

Page 15: 9694 THINKING SKILLS - maxpapersmaxpapers.com/wp-content/uploads/2012/11/9694_w14_ms_complete… · 9694 THINKING SKILLS 9694/12 Paper 1 ... October/November 2014 9694 12 ... 1-mark

Page 7 Mark Scheme Syllabus Paper

Cambridge International AS/A Level – October/November 2014 9694 21

© Cambridge International Examinations 2014

(d) ‘Advertising should be controlled by law.’ Write your own short argument to support or challenge this claim. The conclusion of

your argument must be stated. Credit will not be given for repeating ideas from the passage. [5]

Level 3 4–5 marks

Developed, coherent argument. Reasons strongly support conclusion.

Development may include intermediate conclusion or apt examples.

Simply structured argument – 4 marks.

Effective use of IC etc. – 5 marks.

Level 2 2–3 marks

A simple argument. One reason + conclusion – 2 marks.

Two or more separate reasons + conclusion – 3 marks.

Level 1 1 mark

Some relevant comment.

Level 0 0 marks

No relevant comment.

Maximum 3 marks for wrong conclusion or if conclusion is implied but not stated. No credit for material merely reproduced from the passage.

Specimen level 3 answers

Support [97 words]

There is an imbalance of power between advertisers and the public. Since protecting the

public is the core task of governments, they have a duty to protect people from being misled

by the claims of advertisers.

The only people who are disadvantaged by controls on advertising are companies which

wish to trick people into buying goods or services which are not what they claim to be.

Companies of this kind do not deserve to be protected, and there is therefore no good

reason to allow dishonest advertising to take place.

Therefore advertising should be controlled by law.

Challenge [94 words]

Sane adults have the right to make their own choices and to spend their money however

they think fit. Because controls would restrict the information made available to them, they

would compromise their freedom of choice.

Controls on advertising impose a further burden on manufacturers and retailers, who are

already hampered by laws regarding safety at work, employment rights and a host of other

topics. It is the duty of governments to promote economic growth. So they should remove as

many of these restrictions as possible.

Therefore advertising should not be controlled by law.

Page 16: 9694 THINKING SKILLS - maxpapersmaxpapers.com/wp-content/uploads/2012/11/9694_w14_ms_complete… · 9694 THINKING SKILLS 9694/12 Paper 1 ... October/November 2014 9694 12 ... 1-mark

® IGCSE is the registered trademark of Cambridge International Examinations.

CAMBRIDGE INTERNATIONAL EXAMINATIONS

Cambridge International Advanced Subsidiary and Advanced Level

MARK SCHEME for the October/November 2014 series

9694 THINKING SKILLS

9694/22 Paper 2 (Critical Thinking), maximum raw mark 45

This mark scheme is published as an aid to teachers and candidates, to indicate the requirements of the examination. It shows the basis on which Examiners were instructed to award marks. It does not indicate the details of the discussions that took place at an Examiners’ meeting before marking began, which would have considered the acceptability of alternative answers. Mark schemes should be read in conjunction with the question paper and the Principal Examiner Report for Teachers. Cambridge will not enter into discussions about these mark schemes. Cambridge is publishing the mark schemes for the October/November 2014 series for most Cambridge IGCSE

®, Cambridge International A and AS Level components and some

Cambridge O Level components.

Page 17: 9694 THINKING SKILLS - maxpapersmaxpapers.com/wp-content/uploads/2012/11/9694_w14_ms_complete… · 9694 THINKING SKILLS 9694/12 Paper 1 ... October/November 2014 9694 12 ... 1-mark

Page 2 Mark Scheme Syllabus Paper

Cambridge International AS/A Level – October/November 2014 9694 22

© Cambridge International Examinations 2014

1 (a) What is the significance of the evidence in Source A that the rescue services were not alerted until 2 hours after the party was due back? [3]

Had the rescue services been alerted much earlier the children might not have been in such

a poor state and the incident far less serious [1]. The delay suggests either an attempt at a

cover-up by the centre [1] or incompetence in not alerting the rescue services earlier [1]. It is

consistent with the evidence in Source B that the safety standards were inadequate [1]. Lack

of flares/radio meant that the canoe party was unable to alert rescuers themselves [1].

(b) How useful is the evidence given in Source B in judging responsibility for the canoeing incident? [3]

It is from an expert source [1] on the inside of the company [1]. We have no reason to

believe that the instructor has a motive beyond expressing serious concerns about safety [1]. Because the memo is before the incident there is no vested interest for the instructor to

exculpate himself [1].

However, Source B concerns matters such as poor equipment and insufficient training for the

children, which may not be very relevant as the main cause of the incident seems to have

been the poor decision to ‘abandon’ the children [1]. The information about staff resignations

is not useful as we have no information as to whether this was due to concerns about safety

[1]. Also the information may no longer be relevant, because the centre may have responded

to the letter by bringing safety levels up to standard before the incident in March [1].

Maximum 2 marks if one side considered. No mark for judgment. (c) How relevant is the evidence in Source E in coming to a judgement about

responsibility for the incident? [3] The first part of the evidence is not relevant [1] as the children’s mental and physical state

would have been the result of what had happened to them irrespective of who was to blame

[1].

However, the second part is relevant [1] as it suggests the centre had been unwise to let the

children go out to sea [1]. The possible defence that conditions had suddenly changed is

undermined by the fact that the wind had been blowing offshore for several days [1].

Do not allow points about reliability of evidence, e.g. from an eye witness. Maximum 2 marks if only one side considered.

Page 18: 9694 THINKING SKILLS - maxpapersmaxpapers.com/wp-content/uploads/2012/11/9694_w14_ms_complete… · 9694 THINKING SKILLS 9694/12 Paper 1 ... October/November 2014 9694 12 ... 1-mark

Page 3 Mark Scheme Syllabus Paper

Cambridge International AS/A Level – October/November 2014 9694 22

© Cambridge International Examinations 2014

(d) Should the Budmouth Sea Adventure Centre be held responsible for the sea-canoeing incident? Write a short, reasoned argument to support your conclusion, with critical reference to the evidence provided and considering a plausible alternative conclusion.

[6]

Level 3 5–6 marks

A strong answer, which provides a reasoned argument including thorough

evaluation of all or most of the evidence to support an acceptable conclusion

in terms of probability and evaluates the plausibility of at least one alternative

conclusion.

Level 2 3–4 marks

An answer which evaluates some of the evidence, draws an acceptable

conclusion in terms of probability and may mention the plausibility of at least

one alternative conclusion.

Level 1 1–2 marks

A weak answer, which refers to some of the evidence, possibly including a

simple evaluative comment. The conclusion may be unstated or over-stated.

Level 0 0 marks

No credit-worthy material.

Indicative Content Possible answers:

• The accident was caused by negligence on the part of the centre, e.g. in

underestimating dangers involved in sea canoeing as indicated in Source D.

• The accident was caused by negligence on the part of the instructors, e.g.

mishandling of situation suggested in Source A.

• The accident was simply due to changes in the weather etc. and the inherent risk in

any ‘adventure’ activity e.g. if the risk of sea canoeing was as great as alleged in

Source D it is unlikely that the activity would be allowed. Given their public profile it

would seem unlikely the Centre is engaged in an illegal activity.

• The accident was due to the centre being misinformed about the level of expertise of

the participants e.g. the centre had said that participants needed 30 hours of

canoeing experience in a freshwater environment. This line of thought is rather

speculative but should be credited if used as a ‘plausible alternative conclusion’.

The evidence against the Budmouth Sea Adventure Centre is rather damning.

Source A suggests that the original incident was rather mishandled with the children drifting

off with none of the adults being aware of this. In addition, they lacked flares and the incident

was not reported promptly to the rescue services.

Source B which is an expert source inside the company suggests that equipment and

preparation were probably inadequate.

The claim in Source C that the environment is ‘ultimately safe’ is undermined by the expert

evidence in Source D. It seems unlikely that the children had sufficient freshwater experience

and the sea would seem to be an inherently unsafe environment.

Source E suggests weather and tide conditions were not suitable for the expedition and the

centre should have known this.

Page 19: 9694 THINKING SKILLS - maxpapersmaxpapers.com/wp-content/uploads/2012/11/9694_w14_ms_complete… · 9694 THINKING SKILLS 9694/12 Paper 1 ... October/November 2014 9694 12 ... 1-mark

Page 4 Mark Scheme Syllabus Paper

Cambridge International AS/A Level – October/November 2014 9694 22

© Cambridge International Examinations 2014

2 (a) ‘The sample used in the research in Source A is not sufficiently representative to allow its conclusion about the beneficial effects of chocolate to be drawn.’ Is this a valid criticism? [3]

Although the study is confined to men, this is an objection to the range of the research not

the validity of the conclusion [1]. Although it consists of Swedish men there is no reason to

believe that the results cannot be generalised to other men [1] given that they share the

same organic human make-up [1]. In any case, the conclusion does not attempt to

generalise beyond the sample [1].

On the other hand:

We are told lifestyle differences have been accounted for, but we do not know whether this is

true of age and health differences [1]. There may be reasons why you cannot generalise

from Swedish men to all men [1] e.g. genetic differences linked to ethnicity [1].

Allow: Women not included [1] nor are other ethnic groups/nationalities [1].

No judgment mark. (b) If “chocolate is good for you”, can you reliably conclude that “eating a Whizzo! will

benefit your health”? (Source B) [3]

No [1]. The other ingredients in Whizzo! could be bad for you [1] meaning that any beneficial

effects of the chocolate would be outweighed by the bad effects of the other ingredients [1]. Even if one Whizzo! is good for you, eating lots of them may not be [1].

(c) Does the information about comfort food in Source E challenge the conclusion about the health benefits of chocolate in Source A? Justify your answer. [3]

No [1]. Source A draws a conclusion about the benefits of chocolate on physical health [1]. Information about the use of chocolate as a psychological aid does not affect this conclusion

[1]. Also, the gender difference suggests chocolate may be of less significance to men as a

comfort food [1]. Additionally, one would have to make the assumption that it is bad to use

food in this way [1] and that such use is necessarily linked to overindulgence [1].

Page 20: 9694 THINKING SKILLS - maxpapersmaxpapers.com/wp-content/uploads/2012/11/9694_w14_ms_complete… · 9694 THINKING SKILLS 9694/12 Paper 1 ... October/November 2014 9694 12 ... 1-mark

Page 5 Mark Scheme Syllabus Paper

Cambridge International AS/A Level – October/November 2014 9694 22

© Cambridge International Examinations 2014

(d) “Chocolate is good for you.” How justified is this statement? Write a short, reasoned argument to support your

conclusion, using and evaluating the information provided in Sources A–E. [6]

Level 3 5–6 marks

A reasoned argument, which uses and evaluates all or most of the evidence

provided.

Level 2 3–4 marks

A simple argument, which uses and/or evaluates evidence.

Level 1 1–2 marks

A weak answer, which makes some correct reference to evidence but

consists of opinion and/or assertion rather than argument

or a weak argument which makes no reference to evidence.

Level 0 0 marks

No credit-worthy material.

Indicative content

• Source A suggests there are health benefits in moderate consumption for Swedish

men.

• However, this might not apply to other groups/excessive consumption.

• Source B indicates the dangers of health being used to promote chocolate bars.

• Source C directly challenges the statement by showing chocolate to be unhealthy.

• Evidence is from a credible expert.

• However only deals with consumption of large amounts.

• Source D suggests it can be addictive which would encourage excessive

consumption.

• Source E suggests the statement is particularly inapplicable to women who are

more likely to use chocolate as a ‘comfort food’. This is likely to encourage

excessive consumption.

The statement is too sweeping and none of the sources would support the statement in this

particular form.

Page 21: 9694 THINKING SKILLS - maxpapersmaxpapers.com/wp-content/uploads/2012/11/9694_w14_ms_complete… · 9694 THINKING SKILLS 9694/12 Paper 1 ... October/November 2014 9694 12 ... 1-mark

Page 6 Mark Scheme Syllabus Paper

Cambridge International AS/A Level – October/November 2014 9694 22

© Cambridge International Examinations 2014

3 (a) Using the exact words from the passage as far as possible, identify the main conclusion. [2]

2 marks: The electric car will prove a disappointment to those who think it is (a solution to the

problem of the car).

1 mark: This (the electric car) is not the solution to the problem of the car that many hope for.

(b) Using the exact words from the passage as far as possible, identify three reasons used to support the main conclusion. [3]

• This (the electric car) is not the solution to the problem of the car that many hope

for.

• They are no use on a long journey.

• An age of the electric car would not see any major reduction in pollution of the

atmosphere.

• One either has a noisy and safe electric car or a silent and deadly one.

• The electric car makes no contribution to the problem of urban congestion.

(c) Evaluate the strength of the reasoning in the argument. In your answer you should consider any flaws, unstated assumptions and other weaknesses. [5]

Marks for each evaluative point as follows, up to a maximum of 5 marks: 2 marks Valid evaluative point, clearly expressed. 1 mark Weak attempt at a valid evaluative point. Paragraph 2

• Assumption: that many people want/need to use their cars for long journeys.

• Assumption: there are long distances between/infrequent charging points.

• Inconsistency: if the early motor car overcame similar problems it suggests the

electric car could one day overcome them as well.

• Exaggerated conclusion: whilst a long journey may be more difficult and requiring

more planning one cannot conclude they are of ‘no use’. Even as regards difficulty,

we cannot conclude this without more information about re-charging points etc.

Paragraph 3

• Assumption: that producing electricity necessarily involves burning fossil fuels.

• One cannot draw the conclusion that there would not be a major reduction in

pollution without information about the relative amount of pollution. Producing

electricity to power cars, even using fossil fuels, may require less consumption of

fossil fuel/produce less pollution than that required to power the individual fossil-

fuelled car.

Page 22: 9694 THINKING SKILLS - maxpapersmaxpapers.com/wp-content/uploads/2012/11/9694_w14_ms_complete… · 9694 THINKING SKILLS 9694/12 Paper 1 ... October/November 2014 9694 12 ... 1-mark

Page 7 Mark Scheme Syllabus Paper

Cambridge International AS/A Level – October/November 2014 9694 22

© Cambridge International Examinations 2014

Paragraph 4

• Assumption: that the electric car has to be noisy all the time to be safe for

pedestrians/cyclists. There could be some sort of automatic warning that sounded

when the car was approaching a pedestrian or cyclist.

• Assumption: that there are not already very quiet and dangerous hazards; cycles

already present such a danger to pedestrians.

• Assumption: other road users rely on hearing to become aware of hazards.

• The noisy electric car would still fulfil the main aim of a less polluting/fossil fuel using

means of transport.

• Restricting the options: it might be possible to create a noise that is sufficient to

warn pedestrians and cyclists which is still not as noisy as the internal combustion

engine. Electric cars could therefore be considerably quieter.

• “Deadly” may exaggerate the danger posed by noiseless vehicles.

Paragraph 5

• Assumption: that congestion caused is the only factor that needs considering when

evaluating traffic jams. Stationary traffic also increases pollution therefore a traffic

jam of electric cars would not be the same as a traffic jam of fossil-fuelled cars as it

would cause less pollution. One could use a similar argument as regards noise.

• The electric car would not have to solve all problems associated with cars in order

not to prove a disappointment.

Page 23: 9694 THINKING SKILLS - maxpapersmaxpapers.com/wp-content/uploads/2012/11/9694_w14_ms_complete… · 9694 THINKING SKILLS 9694/12 Paper 1 ... October/November 2014 9694 12 ... 1-mark

Page 8 Mark Scheme Syllabus Paper

Cambridge International AS/A Level – October/November 2014 9694 22

© Cambridge International Examinations 2014

(d) ‘It would be better if cars did not exist.’ Write your own short argument to support or challenge this claim. The conclusion of

your argument must be stated. Credit will not be given for repeating ideas from the passage. [5]

Level 3 4–5 marks

Developed, coherent argument. Reasons strongly support conclusion.

Development may include intermediate conclusion or apt examples.

Simply structured argument – 4 marks.

Effective use of IC etc. – 5 marks.

Level 2 2–3 marks

A simple argument. One reason + conclusion – 2 marks.

Two or more separate reasons + conclusion – 3 marks.

Level 1 1 mark

Some relevant comment.

Level 0 0 marks

No relevant comment.

Maximum 3 marks for wrong conclusion or if conclusion is implied but not stated. No credit for material merely reproduced from the passage.

Specimen level 3 answers

Support [86 words] Whilst the car is convenient for the individual, the health costs are far too high to justify this

convenience. The use of cars for short journeys has made a major contribution to obesity.

Without their cars, people would be far more likely to walk and cycle on short journeys thus

getting the exercise they need to maintain good health. Obesity is a major contributor to

conditions such as heart disease and diabetes which shorten life expectancy. So it would be

better if cars did not exist.

Challenge [103 words] The car has been a major contributor to the freedom of the individual. Before the car, people

lived very narrow and boring lives rarely going outside their immediate neighbourhoods.

Some people lived their whole lives never venturing further than a few miles from their

homes. With the car, people were able to experience a great variety of different places and

so broaden their horizons. It may well be that the liberation the car brought was a major

contributor to creating the questioning mind-set that is a key requirement of modern

democratic society. So it would not be better if cars did not exist.

Page 24: 9694 THINKING SKILLS - maxpapersmaxpapers.com/wp-content/uploads/2012/11/9694_w14_ms_complete… · 9694 THINKING SKILLS 9694/12 Paper 1 ... October/November 2014 9694 12 ... 1-mark

® IGCSE is the registered trademark of Cambridge International Examinations.

CAMBRIDGE INTERNATIONAL EXAMINATIONS

Cambridge International Advanced Subsidiary and Advanced Level

MARK SCHEME for the October/November 2014 series

9694 THINKING SKILLS

9694/23 Paper 2 (Critical Thinking), maximum raw mark 45

This mark scheme is published as an aid to teachers and candidates, to indicate the requirements of the examination. It shows the basis on which Examiners were instructed to award marks. It does not indicate the details of the discussions that took place at an Examiners’ meeting before marking began, which would have considered the acceptability of alternative answers. Mark schemes should be read in conjunction with the question paper and the Principal Examiner Report for Teachers. Cambridge will not enter into discussions about these mark schemes. Cambridge is publishing the mark schemes for the October/November 2014 series for most Cambridge IGCSE

®, Cambridge International A and AS Level components and some

Cambridge O Level components.

Page 25: 9694 THINKING SKILLS - maxpapersmaxpapers.com/wp-content/uploads/2012/11/9694_w14_ms_complete… · 9694 THINKING SKILLS 9694/12 Paper 1 ... October/November 2014 9694 12 ... 1-mark

Page 2 Mark Scheme Syllabus Paper

Cambridge International AS/A Level – October/November 2014 9694 23

© Cambridge International Examinations 2014

1 (a) Suggest two ways in which Source A could be relevant in deciding whether the three Eastland soldiers were guilty of spying in Westland territory. [4]

2 marks for each clear, valid answer. 1 mark for each vague or incomplete answer. 2 answers required.

If Eastland believe that Westland are preparing to attack them, they have a motive for

sending spies into Westland territory.

If Westland are preparing to attack Eastland, they may have kidnapped the soldiers and

accused them of spying in order to give them an excuse for military action.

1 mark for The use of spy satellites makes the use of human spies unnecessary/more likely.

(b) How reliable is Source B in deciding whether the Eastland soldiers were spies? [2]

Unreliable [1]. The Eastland authorities have a strong vested interest to promote an account

of the incident in which their own soldiers are innocent [1], in order to reduce the likelihood of

harm coming to the soldiers [1] and political damage to themselves [1]. The newspaper

would know only what the authorities wanted it to know [1].

(c) How useful is the evidence in Source D in deciding whether the soldiers were spying? [3]

The report is reliable, because the Red Cross has an excellent reputation for neutrality [1]. The captured soldiers’ denials are not very credible, because of their strong vested interest

[1]. The fact that their explanation is inconsistent with the official explanation given by their

own government [1] suggests that neither is true and that the soldiers therefore were spying

[1]. Evidence about how well they are being treated is not relevant to the issue of whether

they have been spying or not [1], except that if Westland authorities really believed they were

spies, they might have been treated less well [1].

No mark for judgment.

Page 26: 9694 THINKING SKILLS - maxpapersmaxpapers.com/wp-content/uploads/2012/11/9694_w14_ms_complete… · 9694 THINKING SKILLS 9694/12 Paper 1 ... October/November 2014 9694 12 ... 1-mark

Page 3 Mark Scheme Syllabus Paper

Cambridge International AS/A Level – October/November 2014 9694 23

© Cambridge International Examinations 2014

(d) Do you think the Eastland soldiers were guilty of spying? Write a short, reasoned argument to support your conclusion, with critical reference to the evidence provided and considering a plausible alternative conclusion. [6]

Level 3 5–6 marks

A strong answer, which provides a reasoned argument including thorough

evaluation of all or most of the evidence to support an acceptable conclusion

in terms of probability and evaluates the plausibility of at least one alternative

conclusion.

Level 2 3–4 marks

An answer which evaluates some of the evidence, draws an acceptable

conclusion in terms of probability and may mention the plausibility of at least

one alternative conclusion.

Level 1 1–2 marks

A weak answer, which refers to some of the evidence, possibly including a

simple evaluative comment. The conclusion may be unstated or over-stated.

Level 0 0 marks

No credit-worthy material.

Indicative content

• As the answer to (c) indicates, it is probable that the men were captured while on a

spying mission.

• The other two possible explanations are that they were kidnapped from their barracks or

accidentally strayed over the border.

These are individually plausible, but each of them is the strongest reason against believing

the other.

Most of the sources have strong bias and vested interest.

The spying equipment displayed on Westland television could have been captured with the

Eastland soldiers or supplied by Westland in order to incriminate them. The interpreters did

not necessarily translate what the captured soldiers actually said.

Page 27: 9694 THINKING SKILLS - maxpapersmaxpapers.com/wp-content/uploads/2012/11/9694_w14_ms_complete… · 9694 THINKING SKILLS 9694/12 Paper 1 ... October/November 2014 9694 12 ... 1-mark

Page 4 Mark Scheme Syllabus Paper

Cambridge International AS/A Level – October/November 2014 9694 23

© Cambridge International Examinations 2014

2 (a) Suggest two reasons why Source B is more reliable than Source A. [3]

3 marks: 2 reasons, at least one of which is developed 2 marks: 1 developed reason or 2 undeveloped reasons 1 mark: 1 undeveloped reason 0 marks: No credit-worthy material Sample developed answers: Both sources are biased, because each gives only one side of the picture, but the claims in

Source A are more extreme, and it is therefore more biased than Source B.

Source A has a clear vested interest (to sell its own alternative to coffee). Source B has no

explicit VI, although it would not be surprising if it were sponsored by the coffee industry.

Source B has better expertise than Source A, because its claims are derived from

professional medical journals, but the use of scientific terminology in Source A suggests it

has some expertise.

Some of the claims in Source B are obscure and implausible, but the claims in Source A are

more extreme, even more obscure and even less plausible. So Source B is more reliable.

Sample undeveloped answers (incomplete/over-stated): Source A has a clear vested interest, to sell its own alternative to coffee.

Source A has a clear vested interest, to sell its own alternative to coffee, but Source B has

no VI.

Source A is biased, because it mentions only the negative aspects of coffee.

The claims in Source A are obscure and implausible.

(b) Is Source C an argument? Briefly explain your answer. [2]

2 marks for a correct answer with an accurate explanation. 1 mark for a correct answer with a vague, incomplete or generic explanation. 0 marks for a correct answer without an explanation. 0 marks for an incorrect answer with or without an explanation.

2-mark answer Source C is not an argument. It is a factual report on a research project, which does not try to

persuade the reader to do anything.

1-mark answers

Source C is not an argument, because it does not include a persuasive conclusion.

Source C is not an argument. It is a factual report on a research project.

Page 28: 9694 THINKING SKILLS - maxpapersmaxpapers.com/wp-content/uploads/2012/11/9694_w14_ms_complete… · 9694 THINKING SKILLS 9694/12 Paper 1 ... October/November 2014 9694 12 ... 1-mark

Page 5 Mark Scheme Syllabus Paper

Cambridge International AS/A Level – October/November 2014 9694 23

© Cambridge International Examinations 2014

(c) Suggest two ways in which the sample used for the research in Source C might not be representative. Briefly explain the significance of your answers. [4]

2 marks for each clear, valid answer. 1 mark for each vague, incomplete or marginal answer.

2 answers required.

The factors which may be significant are:

Employment (the participants of the survey were all in a stressful occupation and they were

also likely to be aware of health issues).

Nationality/location (the participants of the survey were all resident in the United States).

Age (the average age of participants is quite high).

Education (the participants of the survey were all educated to a professional level).

Explanations of the possible significance of a factor should refer to the incidence of

coffee-drinking and/or of depression.

Allow 1 mark for: All the participants were female.

Examples of 2-mark answers The participants in the survey were all engaged in a stressful occupation and may therefore

have been more liable to suffer from depression.

The participants in the survey all worked in a medical environment, and may therefore have

been more likely to recognise symptoms of depression and more likely to seek treatment for

it.

Examples of 1-mark answers The participants in the survey were all engaged in a stressful occupation.

The participants in the survey all worked in a medical environment.

Page 29: 9694 THINKING SKILLS - maxpapersmaxpapers.com/wp-content/uploads/2012/11/9694_w14_ms_complete… · 9694 THINKING SKILLS 9694/12 Paper 1 ... October/November 2014 9694 12 ... 1-mark

Page 6 Mark Scheme Syllabus Paper

Cambridge International AS/A Level – October/November 2014 9694 23

© Cambridge International Examinations 2014

(d) ‘Everyone should drink more coffee for the sake of their health.’ To what extent do you agree with this statement? Write a short, reasoned argument to

support your conclusion, using and evaluating the information provided in Sources A–D. [6]

Level 3 5–6 marks

A reasoned argument, which uses and evaluates all or most of the evidence

provided.

Level 2 3–4 marks

A simple argument, which uses and/or evaluates evidence.

Level 1 1–2 marks

A weak answer, which makes some correct reference to evidence but

consists of opinion and/or assertion rather than argument

or a weak argument which makes no reference to evidence.

Level 0 0 marks

No credit-worthy material.

Indicative content

• The claim cannot apply to “everyone”, because some people may already drink enough

coffee, or even too much.

• Source A carries very little weight, because of its obvious strong vested interest and the

implausibility of its claims.

• Source B suggests that drinking coffee has more health benefits than drawbacks, and

thereby implies that many people would benefit to some extent from drinking more

coffee.

• Source B seems to be based on expertise/evidence, but the evidence is one-sided and

presumably selective (biased).

• The research reported in Source C suggests that a fairly low level of coffee drinking has

some benefit to mental health, but the range of the sample was limited.

Page 30: 9694 THINKING SKILLS - maxpapersmaxpapers.com/wp-content/uploads/2012/11/9694_w14_ms_complete… · 9694 THINKING SKILLS 9694/12 Paper 1 ... October/November 2014 9694 12 ... 1-mark

Page 7 Mark Scheme Syllabus Paper

Cambridge International AS/A Level – October/November 2014 9694 23

© Cambridge International Examinations 2014

3 (a) Using the exact words from the passage as far as possible, identify the main conclusion. [2]

2 marks: The only qualification required to be a teacher should (, therefore,) be the ability to

enable students to pass their exams.

1 mark: recognisable paraphrase of the above.

1 mark: (This shows that) possession of a university degree is no guarantee that someone

will be a good teacher.

(b) Using the exact words from the passage as far as possible, identify three reasons

used to support the main conclusion. [3]

• (This shows that) possession of a university degree is no guarantee that someone will be

a good teacher.

• Under the present system, they [degrees] mean nothing.

• Experts are often bad teachers.

• (Any teacher knows) that [teachers need qualifications in order to show that they have

been taught how to teach] is not true.

• Such a change would also be consistent with current trends.

(c) Evaluate the strength of the reasoning in the argument. In your answer you should

consider any flaws, unstated assumptions and other weaknesses. [5]

Marks for each evaluative point as follows, up to a maximum of 5 marks: 2 marks Valid evaluative point, clearly expressed. 1 mark Weak attempt at a valid evaluative point.

Paragraph 1

• The key flaw in this argument is false dichotomy (restricted options). The choice stated

here is not very realistic, since most teachers have both qualifications and at least some

interest in their subject.

• The fact that a degree is not sufficient to make someone a good teacher does not mean

it is not necessary.

• The intermediate conclusion is quite moderate, but the author subsequently interprets it

as if it had been much stronger. Although possession of a degree does not guarantee

that the graduate will be a good teacher, it makes it probable that he/she will be

reasonably competent – or, at least, more competent than he/she would have been

without a degree.

Paragraph 2

• The intermediate conclusion is over-stated. The argument would support only a much

weaker version of this conclusion.

Paragraph 3

• Another false dichotomy. Education is about both teaching and learning.

Page 31: 9694 THINKING SKILLS - maxpapersmaxpapers.com/wp-content/uploads/2012/11/9694_w14_ms_complete… · 9694 THINKING SKILLS 9694/12 Paper 1 ... October/November 2014 9694 12 ... 1-mark

Page 8 Mark Scheme Syllabus Paper

Cambridge International AS/A Level – October/November 2014 9694 23

© Cambridge International Examinations 2014

Paragraph 4

• Also based on a false dichotomy. Teachers are not divided simply into those who can

teach and those who cannot. Rather, all teachers can be located somewhere on a

continuum of competence, and few of them, if any, would not benefit from a course of

training.

• Inconsistency: para 3 claims that education is not about teaching, but para 4 focuses on

teaching.

Paragraph 5

• The two sentences are inconsistent with one another. If qualifications are less important

than skills, then the ability to enable students to gain qualifications (= pass exams)

cannot be the crucial quality for a teacher.

(d) ‘Gaining qualifications is the main purpose of education.’ Write your own short argument to support or challenge this claim. The conclusion of

your argument must be stated. Credit will not be given for repeating ideas from the passage. [5]

Level 3 4–5 marks

Developed, coherent argument. Reasons strongly support conclusion.

Development may include intermediate conclusion or apt examples.

Simply structured argument – 4 marks.

Effective use of IC etc. – 5 marks.

Level 2 2–3 marks

A simple argument. One reason + conclusion – 2 marks.

Two or more separate reasons + conclusion – 3 marks.

Level 1 1 mark

Some relevant comment.

Level 0 0 marks

No relevant comment.

Maximum 3 marks for wrong conclusion or if conclusion is implied but not stated. No credit for material merely reproduced from the passage.

Page 32: 9694 THINKING SKILLS - maxpapersmaxpapers.com/wp-content/uploads/2012/11/9694_w14_ms_complete… · 9694 THINKING SKILLS 9694/12 Paper 1 ... October/November 2014 9694 12 ... 1-mark

Page 9 Mark Scheme Syllabus Paper

Cambridge International AS/A Level – October/November 2014 9694 23

© Cambridge International Examinations 2014

Specimen level 3 answers

Support [97 words] When you are appointing someone to a job, it is not easy to discover whether candidates

have the qualities you are looking for or not. References and interviews are subjective and

may be mistaken. But qualifications are objective and provide a fair basis for differentiating

between candidates. So schools, colleges and universities should prepare students for the

world of work by enabling them to gain qualifications.

Qualifications are also the only reliable way of assessing the ability of students and how hard

they and their teachers have worked.

Gaining qualifications is, therefore, the main purpose of education.

Challenge [103 words] One aspect of education is to enable students to gain the qualifications which indicate to

potential employers that they have the knowledge and skills required to earn a living, but the

knowledge and skills themselves are more important than the piece of paper.

Furthermore, life is more than work, and education for life therefore has more dimensions

than preparation for work. Education should prepare students for intimate relationships and

family life. It should also introduce them to cultural and sporting activities, some of which they

may choose to take with them into adulthood.

Gaining qualifications is, therefore, not the main purpose of education.

Page 33: 9694 THINKING SKILLS - maxpapersmaxpapers.com/wp-content/uploads/2012/11/9694_w14_ms_complete… · 9694 THINKING SKILLS 9694/12 Paper 1 ... October/November 2014 9694 12 ... 1-mark

® IGCSE is the registered trademark of Cambridge International Examinations.

CAMBRIDGE INTERNATIONAL EXAMINATIONS

Cambridge International Advanced Level

MARK SCHEME for the October/November 2014 series

9694 THINKING SKILLS

9694/31 Paper 3 (Problem Analysis and Solution), maximum raw mark 50

This mark scheme is published as an aid to teachers and candidates, to indicate the requirements of the examination. It shows the basis on which Examiners were instructed to award marks. It does not indicate the details of the discussions that took place at an Examiners’ meeting before marking began, which would have considered the acceptability of alternative answers. Mark schemes should be read in conjunction with the question paper and the Principal Examiner Report for Teachers. Cambridge will not enter into discussions about these mark schemes. Cambridge is publishing the mark schemes for the October/November 2014 series for most Cambridge IGCSE

®, Cambridge International A and AS Level components and some

Cambridge O Level components.

Page 34: 9694 THINKING SKILLS - maxpapersmaxpapers.com/wp-content/uploads/2012/11/9694_w14_ms_complete… · 9694 THINKING SKILLS 9694/12 Paper 1 ... October/November 2014 9694 12 ... 1-mark

Page 2 Mark Scheme Syllabus Paper

Cambridge International A Level – October/November 2014 9694 31

© Cambridge International Examinations 2014

1 (a) (i) Which of these squares might be found in a different position after shipping? [2]

1 and 2 (1 mark for both) and 4 (1 mark)

(ii) Draw a rearrangement of these pieces inside a 5 × 12 rectangle which would result in fewer pieces being able to move. [1]

(b) How many unit squares would be needed to fill all the gaps? [1]

There is no requirement to find the arrangement.

19 × 27 = 513

1 + 4 + 9 + 16 + 25 + 36 + 49 + 64 + 81 + 100 + 121 = 506

513 – 506 = 7

(c) Which one of these seven squares can never move, no matter how many of the others do? [1]

Only the 5 by 5 (bottom right hand) is stuck.

2

3

4 5

1

2

34 5

1

34 51

67

2

Page 35: 9694 THINKING SKILLS - maxpapersmaxpapers.com/wp-content/uploads/2012/11/9694_w14_ms_complete… · 9694 THINKING SKILLS 9694/12 Paper 1 ... October/November 2014 9694 12 ... 1-mark

Page 3 Mark Scheme Syllabus Paper

Cambridge International A Level – October/November 2014 9694 31

© Cambridge International Examinations 2014

(d) Design such a ‘filler’ piece, and show where the smallest square should be placed relative to it. [2]

Examples of possible shapes are shown below. Award 1 mark for an appropriate filler, and a

further mark for the placement of the smallest square.

If 2 marks cannot be given, award one for an arrangement which allows only one item to

move, or the six units are not used as a single piece, or it uses 7 units.

(e) Draw another arrangement of the seven squares, without any extra pieces, within this

11 × 14 rectangle, so that none of the squares bigger than 3 × 3 can move. [3]

Various arrangements are possible, and need to check only 1 × 1, 2 × 2, 3 × 3 move e.g.

Allow 2 marks if one larger square can still move.

If 2 marks cannot be awarded, allow 1 mark for an arrangement in which two pieces are fixed

OR the 7×7 square is fixed OR an arrangement using a 22×7 rectangle.

234 51

67

234 5

1

67

2 34 5

167

234 5

167

2

3

4

5

1

7 6

Page 36: 9694 THINKING SKILLS - maxpapersmaxpapers.com/wp-content/uploads/2012/11/9694_w14_ms_complete… · 9694 THINKING SKILLS 9694/12 Paper 1 ... October/November 2014 9694 12 ... 1-mark

Page 4 Mark Scheme Syllabus Paper

Cambridge International A Level – October/November 2014 9694 31

© Cambridge International Examinations 2014

2 (a) What 4-digit PIN would 8 7 + produce? [1]

8 5 2 9

(b) How many different 4-digit PINs can be produced using two digits and an addition sign

in this way? [1]

100

(c) What rule would produce the PIN 2 6 4 2? [1]

2 8 –

(d) List all the rules that would produce the PIN 6 6 6 6. [2]

6 0 +, 6 0 –, 6 1 ×, 6 6 ×

1 mark for two correct solutions

(e) In this part, consider only PINs with four different digits. Give an example of such a PIN which can be produced using two different rules, both using multiplication. State the rules. [2]

2 2 × and 2 7 × give 2 4 8 6

4 2 × and 4 7 × give 4 8 6 2

6 2 × and 6 7 × give 6 2 4 8

8 2 × and 8 7 × give 8 6 2 4

2 3 × and 2 8 × give 2 6 8 4

4 3 × and 4 8 × give 4 2 6 8

6 3 × and 6 8 × give 6 8 4 2

8 3 × and 8 8 × give 8 4 2 6

Award 2 marks for two correct rules – even if the code is not stated.

Award 1 mark for a code on its own.

(f) List all of the 4-digit PINs of the form 3 1 _ _ which would not be allowed (i.e. are produced by one of the possible rules)? [2]

3 1 7 9, 3 1 1 3, 3 1 5 9, 3 1 9 7

1 mark for any two of these

(g) Show that at least 97% of all possible 4-digit PINs are still allowed. [1]

The PIN-cracking program cannot produce more than (10 × 10 × 3) out of 10 000 PINs.

Page 37: 9694 THINKING SKILLS - maxpapersmaxpapers.com/wp-content/uploads/2012/11/9694_w14_ms_complete… · 9694 THINKING SKILLS 9694/12 Paper 1 ... October/November 2014 9694 12 ... 1-mark

P

3

Pag

(

(

(

ge

(a)

(b)

(c)

5

(

(

(

) (

(

( (

(i)

ii)

iii)

(i)

ii)

(i)

ii)

C

Go

(1

W

(0

W

H

2

Hso

Tlo

In

afafafaf

S

If co

Dcopth

F

1

Cam

Givene

1, 1

Whe

0.8

Whe

, 5

1

How

× 2

Howom

heowe

nto

ftefteftefte

o t

2 orr

Drawouos

he

or

ma

mb

e te ro

1) O

ere

, 0

ere

5

4

w m

2 ×

w mme

poest

o ho

r 1r 2r 3r 4

the

marect

w ald

sitiosin

ex

ark

rid

heoll-

OR

e d

.05

e d

ma

× 2

masp

oint la

ow

st rnd rrd rth r

e bu

arkt a

a pco

on ng

xam

k fo

dge

e co-ou

R (0

oe

5)

oe

ny

=

nypice

ts ye

w m

roll rolrollroll

utte

s cna

paiom

ofle c

mpl

or c

e In

oorut.

0, 0

es t

es t

y la

8

y roe?

ner is

man

oul ou ouou

er w

canlys

r obinf thco

e:

cor

nte

rdi

0)

the

the

ye

oll-

ar s y

ny

ut : ut ut :ut :

wil

nnosis

of dne he mb

rrec

ern

©

ina

e p

e p

ers

ou

the=

pie

0.: 0 0. 0.

l ha

ot b(w

diatotwbin

ct d

nati

Ca

ates

poin

poin

of

uts

e b1/8

ece

2 –.4 –8 –6 –

ave

be ith

agr fo

wo lned

dia

ion

amb

s o

nt

nt

f gr

ar

bott8, b

es

– 0– 1– 1– 1

e b

awaw

amormlumd lu

agra

M

nal

brid

of a

(0.

(5

3

rou

re n

tombut

wi

0.81 & & &

bee

wardwar

ms m ompum

am

Mar

A

dge

a p

4,

5

3,

und

nee

m waf

ill t

& 0 0 –0 –

en

deren

to ones b

mp

m be

k S

Le

e In

poi

0.1

) m

d s

ede

will fter

the

– 0– 1– 1

cut

d, nes

she lubefaft

efo

Sch

eve

nter

nt

1) m

mo

spi

ed

ner fo

e b

0.61 &1 &

t in

awss o

howumforter

ore

hem

el –

rnat

th

mo

ove

ce

be

eedour

butt

6 & 0 & 0

nto

warof l

w hp d

re tth

, 1

me

– O

tion

at

ove

e to

e w

efo

d a we

ter

– – 1

6

d 1len

howduthe

he r

m

e

Oct

nal

en

e to

o a

ill

ore

laye h

r ha

1 &1 &

pie

1 mngth

w trin

e rorol

ark

tob

Ex

nds

o a

fte

the

e al

yerhav

ave

& 0& 0

ece

marhs)

twong oll-l-o

k fo

ber

xam

s u

afte

er a

ere

ll p

r atve s

e b

– –

es

rk f) up

o la r-ou

out

or m

r/No

mina

p i

er a

a ro

e b

poi

t ospi

bee

0.21 &

for p to

umrollut, .

ma

ov

atio

in t

a r

oll-

be a

nts

r bce

en

2 & 0

woo t

mpsl-oan

atch

vem

ons

the

roll

-ou

afte

s a

eloat

cu

0 –

orkhe

s out.

nd t

hin

mbe

s 20

e s

l-o

ut?

er

are

ow y =

ut a

1 &

kingen

of b. Othe

ng d

er

014

am

ut?

?

thr

wi

y == 1

afte

& 0

g wnd

butOnee o

dia

20

4

me

?

ree

ith

= 11/1

er t

0 –

withof

ttee doth

agra

014

pla

e ro

in

1/106.

the

– 0.

h onthe

r, oiager

am

4

ace

oll

1/1

0. A

e fo

.4

ne e 2

of agradia

m af

S

e a

-ou

10

Aft

ou

ar2nd

anam ag

fter

Syl

9

as

uts

of

ter

rth

rithrol

y sshram

rwa

llab

969

it s

s?

f a

3 r

h ro

mel o

simhoum s

ard

bu

94

sta

un

roll

oll-

eticut.

mpuldsh

ds.

s

arte

nit f

l ou

-ou

c er

le sd sou

P

ed,

fro

uts

ut?

rro

shho

uld

Pa

3

, af

om

s th

?

r O

apow

sh

pe

31

fte

he

OR

pe, the

how

er

r [1

[1

[1

[1

[1

[2

a

e w [2

]

]

]

]

]

2]

2]

Page 38: 9694 THINKING SKILLS - maxpapersmaxpapers.com/wp-content/uploads/2012/11/9694_w14_ms_complete… · 9694 THINKING SKILLS 9694/12 Paper 1 ... October/November 2014 9694 12 ... 1-mark

Page 6 Mark Scheme Syllabus Paper

Cambridge International A Level – October/November 2014 9694 31

© Cambridge International Examinations 2014

(d) (i) How many roll-outs in total are needed before the point (1/7, 4/7) returns to where it started? List all the points in the cycle. [2]

Cycle is (1/7, 4/7) (2/7, 2/7) (4/7, 1/7) [1 mark]

3 roll-outs needed [1 mark]

(ii) Give an example of a point on a different cycle of the same length. (This cycle must not include (1/7, 4/7).) [1]

Any one of (6/7, 3/7), (5/7, 5/7) and (3/7, 6/7). Allow more than one of these but nothing

else.

(e) (i) How many roll-outs in total are needed before the point (1/127, 64/127) returns to where it started? [1]

7

(ii) Identify a point which moves back to its starting position after 2 roll-outs. [1]

(1/3, 2/3) or (2/3, 1/3) (allow both)

(iii) Identify a point which moves back to its starting position after 5 roll-outs. [1]

Any of (1/31, 16/31) (2/31, 8/31) (4/31, 4/31) (8/31, 2/31) (16/31, 1/31) or any

component-wise sum of these, such as (5/31, 20/31).

4 (a) During the festival, which play will be performed

(i) more times than any of the others? [1]

The Tempest (11 performances)

(ii) fewer times than any of the others? [1]

Timon of Athens (3 performances)

The others are as follows:

As You Like It, Twelfth Night, Measure for Measure – 10 each

Romeo and Juliet, Othello – 9 each

Love’s Labour’s Lost – 5

King Lear, Cymbeline – 4 each

(b) Which two dates repeat the schedule of 11 July? [2]

The scheduled plays for these dates are As You Like It, Othello and Measure for Measure.

16 July (accept Tuesday Week 3) [1 mark]

21 July (accept Sunday Week 3) [1 mark]

Page 39: 9694 THINKING SKILLS - maxpapersmaxpapers.com/wp-content/uploads/2012/11/9694_w14_ms_complete… · 9694 THINKING SKILLS 9694/12 Paper 1 ... October/November 2014 9694 12 ... 1-mark

Page 7 Mark Scheme Syllabus Paper

Cambridge International A Level – October/November 2014 9694 31

© Cambridge International Examinations 2014

(c) (i) List all the dates on which Kate will watch either Mark or Antony performing at the festival. [2]

They both perform on 9 July / Tuesday Week 2 and 25 July / Thursday Week 4.

10 July / Wednesday Week 2 (Antony)

15 July / Monday Week 3 (Antony)

17 July / Wednesday Week 3 (Mark)

20 July / Saturday Week 3 (Antony)

23 July / Tuesday Week 4 (Mark)

Award 1 mark for three or four correct dates and/or no more than one incorrect date.

(ii) What is the total cost of Kate’s tickets? [2]

She will miss both opening nights because they clash.

1 × $18 + 4 × $24 = $114

Award 1 mark for evidence of appreciation of 1 ticket @ $18 (Week 2) OR 4 tickets @

$24 (Weeks 3 and 4).

If one or more dates are missing or incorrect in (i), allow 1 follow through mark in (ii) if

the costs are unambiguous and appropriate.

(d) What is the lowest possible total price that he could pay to see all 10 plays? [3]

6 × $15 + 3 × $18 + 1 × $24 = $168

Award 2 marks for 6 @ $15, 3 @ $18 and 1 @ $24 incorrectly totalled, or not totalled.

OR award 1 mark each for evidence of appreciation of the following:

• There are 6 evenings on which (one or more) first performances occur;

• (It is not possible to see both Timon of Athens and Cymbeline during weeks 1 and 2,

so) either Timon of Athens or Cymbeline must be seen during week 3 or week 4.

SC : award 1 mark for one incorrect categorization of play (e.g. 5@15, 4@18, 1@ 24 = $171)

Page 40: 9694 THINKING SKILLS - maxpapersmaxpapers.com/wp-content/uploads/2012/11/9694_w14_ms_complete… · 9694 THINKING SKILLS 9694/12 Paper 1 ... October/November 2014 9694 12 ... 1-mark

Page 8 Mark Scheme Syllabus Paper

Cambridge International A Level – October/November 2014 9694 31

© Cambridge International Examinations 2014

(e) (i) Explain why Richard must go to see Timon of Athens first (on 18 July)? [1]

If he went on 24 July, he could only see King Lear by going to Corioli Park on

consecutive evenings.

(ii) In what order will Richard see the 10 plays? [3]

(Timon of Athens)

The Tempest

Romeo and Juliet

As You Like It

Cymbeline

King Lear

Measure for Measure

Love’s Labour’s Lost

Othello

(Twelfth Night)

Deduct 1 mark:

for each duplication/omission of play seen

for each repetition of venue

if Twelfth Night is not seen last

if two plays’ dates have been swapped.

• The Tempest must be 19 July / Friday Week 3 / second (because he will have gone

to Corioli Park the previous evening to see Timon of Athens, and he is leaving

Twelfth Night until last).

• Measure for Measure must be 24 July / Wednesday Week 4 / seventh (because

Timon of Athens is first and he is leaving Twelfth Night until last).

• Cymbeline must be 22 July / Monday Week 4 / fifth (because Cymbeline on 25 July /

Thursday Week 4 would mean going again to Elsinore Common the evening after

Measure for Measure).

• King Lear must be 23 July / Tuesday Week 4 / sixth (because the dates for Twelfth

Night and The Tempest have already been decided).

• Love’s Labour’s Lost must be 25 July / Thursday Week 4 / eighth (because the dates

for Cymbeline and King Lear have already been decided).

• Romeo and Juliet must be 20 July / Saturday Week 3 / third (because the dates for

Measure for Measure and Love’s Labour’s Lost have already been decided).

• As You Like It must be 21 July / Sunday Week 3 / fourth (because the date for

Measure for Measure has already been decided, and he will have gone to Corioli

Park the previous evening to see Romeo and Juliet).

• Othello must be 26 July / Friday Week 4 / ninth (because the date for The Tempest

has already been decided, and he will have gone to Belmont Gardens the previous

evening to see Love’s Labour’s Lost).

Page 41: 9694 THINKING SKILLS - maxpapersmaxpapers.com/wp-content/uploads/2012/11/9694_w14_ms_complete… · 9694 THINKING SKILLS 9694/12 Paper 1 ... October/November 2014 9694 12 ... 1-mark

® IGCSE is the registered trademark of Cambridge International Examinations.

CAMBRIDGE INTERNATIONAL EXAMINATIONS

Cambridge International Advanced Level

MARK SCHEME for the October/November 2014 series

9694 THINKING SKILLS

9694/32 Paper 3 (Problem Analysis and Solution), maximum raw mark 50

This mark scheme is published as an aid to teachers and candidates, to indicate the requirements of the examination. It shows the basis on which Examiners were instructed to award marks. It does not indicate the details of the discussions that took place at an Examiners’ meeting before marking began, which would have considered the acceptability of alternative answers. Mark schemes should be read in conjunction with the question paper and the Principal Examiner Report for Teachers. Cambridge will not enter into discussions about these mark schemes. Cambridge is publishing the mark schemes for the October/November 2014 series for most Cambridge IGCSE

®, Cambridge International A and AS Level components and some

Cambridge O Level components.

Page 42: 9694 THINKING SKILLS - maxpapersmaxpapers.com/wp-content/uploads/2012/11/9694_w14_ms_complete… · 9694 THINKING SKILLS 9694/12 Paper 1 ... October/November 2014 9694 12 ... 1-mark

Page 2 Mark Scheme Syllabus Paper

Cambridge International A Level – October/November 2014 9694 32

© Cambridge International Examinations 2014

1 (a) (i) Which of these squares might be found in a different position after shipping? [2]

1 and 2 (1 mark for both) and 4 (1 mark)

(ii) Draw a rearrangement of these pieces inside a 5 × 12 rectangle which would result in fewer pieces being able to move. [1]

(b) How many unit squares would be needed to fill all the gaps? [1]

There is no requirement to find the arrangement.

19 × 27 = 513

1 + 4 + 9 + 16 + 25 + 36 + 49 + 64 + 81 + 100 + 121 = 506

513 – 506 = 7

(c) Which one of these seven squares can never move, no matter how many of the others do? [1]

Only the 5 by 5 (bottom right hand) is stuck.

2

3

4 5

1

2

34 5

1

34 51

67

2

Page 43: 9694 THINKING SKILLS - maxpapersmaxpapers.com/wp-content/uploads/2012/11/9694_w14_ms_complete… · 9694 THINKING SKILLS 9694/12 Paper 1 ... October/November 2014 9694 12 ... 1-mark

Page 3 Mark Scheme Syllabus Paper

Cambridge International A Level – October/November 2014 9694 32

© Cambridge International Examinations 2014

(d) Design such a ‘filler’ piece, and show where the smallest square should be placed relative to it. [2]

Examples of possible shapes are shown below. Award 1 mark for an appropriate filler, and a

further mark for the placement of the smallest square.

If 2 marks cannot be given, award one for an arrangement which allows only one item to

move, or the six units are not used as a single piece, or it uses 7 units.

(e) Draw another arrangement of the seven squares, without any extra pieces, within this

11 × 14 rectangle, so that none of the squares bigger than 3 × 3 can move. [3]

Various arrangements are possible, and need to check only 1 × 1, 2 × 2, 3 × 3 move e.g.

Allow 2 marks if one larger square can still move.

If 2 marks cannot be awarded, allow 1 mark for an arrangement in which two pieces are fixed

OR the 7×7 square is fixed OR an arrangement using a 22×7 rectangle.

234 51

67

234 5

1

67

2 34 5

167

234 5

167

2

3

4

5

1

7 6

Page 44: 9694 THINKING SKILLS - maxpapersmaxpapers.com/wp-content/uploads/2012/11/9694_w14_ms_complete… · 9694 THINKING SKILLS 9694/12 Paper 1 ... October/November 2014 9694 12 ... 1-mark

Page 4 Mark Scheme Syllabus Paper

Cambridge International A Level – October/November 2014 9694 32

© Cambridge International Examinations 2014

2 (a) What 4-digit PIN would 8 7 + produce? [1]

8 5 2 9

(b) How many different 4-digit PINs can be produced using two digits and an addition sign

in this way? [1]

100

(c) What rule would produce the PIN 2 6 4 2? [1]

2 8 –

(d) List all the rules that would produce the PIN 6 6 6 6. [2]

6 0 +, 6 0 –, 6 1 ×, 6 6 ×

1 mark for two correct solutions

(e) In this part, consider only PINs with four different digits. Give an example of such a PIN which can be produced using two different rules, both using multiplication. State the rules. [2]

2 2 × and 2 7 × give 2 4 8 6

4 2 × and 4 7 × give 4 8 6 2

6 2 × and 6 7 × give 6 2 4 8

8 2 × and 8 7 × give 8 6 2 4

2 3 × and 2 8 × give 2 6 8 4

4 3 × and 4 8 × give 4 2 6 8

6 3 × and 6 8 × give 6 8 4 2

8 3 × and 8 8 × give 8 4 2 6

Award 2 marks for two correct rules – even if the code is not stated.

Award 1 mark for a code on its own.

(f) List all of the 4-digit PINs of the form 3 1 _ _ which would not be allowed (i.e. are produced by one of the possible rules)? [2]

3 1 7 9, 3 1 1 3, 3 1 5 9, 3 1 9 7

1 mark for any two of these

(g) Show that at least 97% of all possible 4-digit PINs are still allowed. [1]

The PIN-cracking program cannot produce more than (10 × 10 × 3) out of 10 000 PINs.

Page 45: 9694 THINKING SKILLS - maxpapersmaxpapers.com/wp-content/uploads/2012/11/9694_w14_ms_complete… · 9694 THINKING SKILLS 9694/12 Paper 1 ... October/November 2014 9694 12 ... 1-mark

P

3

Pag

(

(

(

ge

(a)

(b)

(c)

5

(

(

(

) (

(

( (

(i)

ii)

iii)

(i)

ii)

(i)

ii)

C

Go

(1

W

(0

W

H

2

Hso

Tlo

In

afafafaf

S

If co

Dcopth

F

1

Cam

Givene

1, 1

Whe

0.8

Whe

, 5

1

How

× 2

Howom

heowe

nto

ftefteftefte

o t

2 orr

Drawouos

he

or

ma

mb

e te ro

1) O

ere

, 0

ere

5

4

w m

2 ×

w mme

poest

o ho

r 1r 2r 3r 4

the

marect

w ald

sitiosin

ex

ark

rid

heoll-

OR

e d

.05

e d

ma

× 2

masp

oint la

ow

st rnd rrd rth r

e bu

arkt a

a pco

on ng

xam

k fo

dge

e co-ou

R (0

oe

5)

oe

ny

=

nypice

ts ye

w m

roll rolrollroll

utte

s cna

paiom

ofle c

mpl

or c

e In

oorut.

0, 0

es t

es t

y la

8

y roe?

ner is

man

oul ou ouou

er w

canlys

r obinf thco

e:

cor

nte

rdi

0)

the

the

ye

oll-

ar s y

ny

ut : ut ut :ut :

wil

nnosis

of dne he mb

rrec

ern

©

ina

e p

e p

ers

ou

the=

pie

0.: 0 0. 0.

l ha

ot b(w

diatotwbin

ct d

nati

Ca

ates

poin

poin

of

uts

e b1/8

ece

2 –.4 –8 –6 –

ave

be ith

agr fo

wo lned

dia

ion

amb

s o

nt

nt

f gr

ar

bott8, b

es

– 0– 1– 1– 1

e b

awaw

amormlumd lu

agra

M

nal

brid

of a

(0.

(5

3

rou

re n

tombut

wi

0.81 & & &

bee

wardwar

ms m ompum

am

Mar

A

dge

a p

4,

5

3,

und

nee

m waf

ill t

& 0 0 –0 –

en

deren

to ones b

mp

m be

k S

Le

e In

poi

0.1

) m

d s

ede

will fter

the

– 0– 1– 1

cut

d, nes

she lubefaft

efo

Sch

eve

nter

nt

1) m

mo

spi

ed

ner fo

e b

0.61 &1 &

t in

awss o

howumforter

ore

hem

el –

rnat

th

mo

ove

ce

be

eedour

butt

6 & 0 & 0

nto

warof l

w hp d

re tth

, 1

me

– O

tion

at

ove

e to

e w

efo

d a we

ter

– – 1

6

d 1len

howduthe

he r

m

e

Oct

nal

en

e to

o a

ill

ore

laye h

r ha

1 &1 &

pie

1 mngth

w trin

e rorol

ark

tob

Ex

nds

o a

fte

the

e al

yerhav

ave

& 0& 0

ece

marhs)

twong oll-l-o

k fo

ber

xam

s u

afte

er a

ere

ll p

r atve s

e b

– –

es

rk f) up

o la r-ou

out

or m

r/No

mina

p i

er a

a ro

e b

poi

t ospi

bee

0.21 &

for p to

umrollut, .

ma

ov

atio

in t

a r

oll-

be a

nts

r bce

en

2 & 0

woo t

mpsl-oan

atch

vem

ons

the

roll

-ou

afte

s a

eloat

cu

0 –

orkhe

s out.

nd t

hin

mbe

s 20

e s

l-o

ut?

er

are

ow y =

ut a

1 &

kingen

of b. Othe

ng d

er

014

am

ut?

?

thr

wi

y == 1

afte

& 0

g wnd

butOnee o

dia

20

4

me

?

ree

ith

= 11/1

er t

0 –

withof

ttee doth

agra

014

pla

e ro

in

1/106.

the

– 0.

h onthe

r, oiager

am

4

ace

oll

1/1

0. A

e fo

.4

ne e 2

of agradia

m af

S

e a

-ou

10

Aft

ou

ar2nd

anam ag

fter

Syl

9

as

uts

of

ter

rth

rithrol

y sshram

rwa

llab

969

it s

s?

f a

3 r

h ro

mel o

simhoum s

ard

bu

94

sta

un

roll

oll-

eticut.

mpuldsh

ds.

s

arte

nit f

l ou

-ou

c er

le sd sou

P

ed,

fro

uts

ut?

rro

shho

uld

Pa

3

, af

om

s th

?

r O

apow

sh

pe

32

fte

he

OR

pe, the

how

er

r [1

[1

[1

[1

[1

[2

a

e w [2

]

]

]

]

]

2]

2]

Page 46: 9694 THINKING SKILLS - maxpapersmaxpapers.com/wp-content/uploads/2012/11/9694_w14_ms_complete… · 9694 THINKING SKILLS 9694/12 Paper 1 ... October/November 2014 9694 12 ... 1-mark

Page 6 Mark Scheme Syllabus Paper

Cambridge International A Level – October/November 2014 9694 32

© Cambridge International Examinations 2014

(d) (i) How many roll-outs in total are needed before the point (1/7, 4/7) returns to where it started? List all the points in the cycle. [2]

Cycle is (1/7, 4/7) (2/7, 2/7) (4/7, 1/7) [1 mark]

3 roll-outs needed [1 mark]

(ii) Give an example of a point on a different cycle of the same length. (This cycle must not include (1/7, 4/7).) [1]

Any one of (6/7, 3/7), (5/7, 5/7) and (3/7, 6/7). Allow more than one of these but nothing

else.

(e) (i) How many roll-outs in total are needed before the point (1/127, 64/127) returns to where it started? [1]

7

(ii) Identify a point which moves back to its starting position after 2 roll-outs. [1]

(1/3, 2/3) or (2/3, 1/3) (allow both)

(iii) Identify a point which moves back to its starting position after 5 roll-outs. [1]

Any of (1/31, 16/31) (2/31, 8/31) (4/31, 4/31) (8/31, 2/31) (16/31, 1/31) or any

component-wise sum of these, such as (5/31, 20/31).

4 (a) During the festival, which play will be performed

(i) more times than any of the others? [1]

The Tempest (11 performances)

(ii) fewer times than any of the others? [1]

Timon of Athens (3 performances)

The others are as follows:

As You Like It, Twelfth Night, Measure for Measure – 10 each

Romeo and Juliet, Othello – 9 each

Love’s Labour’s Lost – 5

King Lear, Cymbeline – 4 each

(b) Which two dates repeat the schedule of 11 July? [2]

The scheduled plays for these dates are As You Like It, Othello and Measure for Measure.

16 July (accept Tuesday Week 3) [1 mark]

21 July (accept Sunday Week 3) [1 mark]

Page 47: 9694 THINKING SKILLS - maxpapersmaxpapers.com/wp-content/uploads/2012/11/9694_w14_ms_complete… · 9694 THINKING SKILLS 9694/12 Paper 1 ... October/November 2014 9694 12 ... 1-mark

Page 7 Mark Scheme Syllabus Paper

Cambridge International A Level – October/November 2014 9694 32

© Cambridge International Examinations 2014

(c) (i) List all the dates on which Kate will watch either Mark or Antony performing at the festival. [2]

They both perform on 9 July / Tuesday Week 2 and 25 July / Thursday Week 4.

10 July / Wednesday Week 2 (Antony)

15 July / Monday Week 3 (Antony)

17 July / Wednesday Week 3 (Mark)

20 July / Saturday Week 3 (Antony)

23 July / Tuesday Week 4 (Mark)

Award 1 mark for three or four correct dates and/or no more than one incorrect date.

(ii) What is the total cost of Kate’s tickets? [2]

She will miss both opening nights because they clash.

1 × $18 + 4 × $24 = $114

Award 1 mark for evidence of appreciation of 1 ticket @ $18 (Week 2) OR 4 tickets @

$24 (Weeks 3 and 4).

If one or more dates are missing or incorrect in (i), allow 1 follow through mark in (ii) if

the costs are unambiguous and appropriate.

(d) What is the lowest possible total price that he could pay to see all 10 plays? [3]

6 × $15 + 3 × $18 + 1 × $24 = $168

Award 2 marks for 6 @ $15, 3 @ $18 and 1 @ $24 incorrectly totalled, or not totalled.

OR award 1 mark each for evidence of appreciation of the following:

• There are 6 evenings on which (one or more) first performances occur;

• (It is not possible to see both Timon of Athens and Cymbeline during weeks 1 and 2,

so) either Timon of Athens or Cymbeline must be seen during week 3 or week 4.

SC : award 1 mark for one incorrect categorization of play (e.g. 5@15, 4@18, 1@ 24 = $171)

Page 48: 9694 THINKING SKILLS - maxpapersmaxpapers.com/wp-content/uploads/2012/11/9694_w14_ms_complete… · 9694 THINKING SKILLS 9694/12 Paper 1 ... October/November 2014 9694 12 ... 1-mark

Page 8 Mark Scheme Syllabus Paper

Cambridge International A Level – October/November 2014 9694 32

© Cambridge International Examinations 2014

(e) (i) Explain why Richard must go to see Timon of Athens first (on 18 July)? [1]

If he went on 24 July, he could only see King Lear by going to Corioli Park on

consecutive evenings.

(ii) In what order will Richard see the 10 plays? [3]

(Timon of Athens)

The Tempest

Romeo and Juliet

As You Like It

Cymbeline

King Lear

Measure for Measure

Love’s Labour’s Lost

Othello

(Twelfth Night)

Deduct 1 mark:

for each duplication/omission of play seen

for each repetition of venue

if Twelfth Night is not seen last

if two plays’ dates have been swapped.

• The Tempest must be 19 July / Friday Week 3 / second (because he will have gone

to Corioli Park the previous evening to see Timon of Athens, and he is leaving

Twelfth Night until last).

• Measure for Measure must be 24 July / Wednesday Week 4 / seventh (because

Timon of Athens is first and he is leaving Twelfth Night until last).

• Cymbeline must be 22 July / Monday Week 4 / fifth (because Cymbeline on 25 July /

Thursday Week 4 would mean going again to Elsinore Common the evening after

Measure for Measure).

• King Lear must be 23 July / Tuesday Week 4 / sixth (because the dates for Twelfth

Night and The Tempest have already been decided).

• Love’s Labour’s Lost must be 25 July / Thursday Week 4 / eighth (because the dates

for Cymbeline and King Lear have already been decided).

• Romeo and Juliet must be 20 July / Saturday Week 3 / third (because the dates for

Measure for Measure and Love’s Labour’s Lost have already been decided).

• As You Like It must be 21 July / Sunday Week 3 / fourth (because the date for

Measure for Measure has already been decided, and he will have gone to Corioli

Park the previous evening to see Romeo and Juliet).

• Othello must be 26 July / Friday Week 4 / ninth (because the date for The Tempest

has already been decided, and he will have gone to Belmont Gardens the previous

evening to see Love’s Labour’s Lost).

Page 49: 9694 THINKING SKILLS - maxpapersmaxpapers.com/wp-content/uploads/2012/11/9694_w14_ms_complete… · 9694 THINKING SKILLS 9694/12 Paper 1 ... October/November 2014 9694 12 ... 1-mark

® IGCSE is the registered trademark of Cambridge International Examinations.

CAMBRIDGE INTERNATIONAL EXAMINATIONS

Cambridge International Advanced Level

MARK SCHEME for the October/November 2014 series

9694 THINKING SKILLS

9694/33 Paper 3 (Problem Analysis and Solution), maximum raw mark 50

This mark scheme is published as an aid to teachers and candidates, to indicate the requirements of the examination. It shows the basis on which Examiners were instructed to award marks. It does not indicate the details of the discussions that took place at an Examiners’ meeting before marking began, which would have considered the acceptability of alternative answers. Mark schemes should be read in conjunction with the question paper and the Principal Examiner Report for Teachers. Cambridge will not enter into discussions about these mark schemes. Cambridge is publishing the mark schemes for the October/November 2014 series for most Cambridge IGCSE

®, Cambridge International A and AS Level components and some

Cambridge O Level components.

Page 50: 9694 THINKING SKILLS - maxpapersmaxpapers.com/wp-content/uploads/2012/11/9694_w14_ms_complete… · 9694 THINKING SKILLS 9694/12 Paper 1 ... October/November 2014 9694 12 ... 1-mark

Page 2 Mark Scheme Syllabus Paper

Cambridge International A Level – October/November 2014 9694 33

© Cambridge International Examinations 2014

1 (a) Give an example of a numbered vertical rock sample involving the numbers 1, 2, 3 and 4 only, without repeating any number, which must have resulted from a fault followed by erosion. [1]

4, 1, 2, 3 OR 3, 4, 1, 2 OR 2, 3, 4, 1

(b) Give an example of such a sample, with at least one number appearing more than once. [1]

1, 2, 1

SC – a sample which was caused by both a fold and a fault (e.g. 1232112321)

(c) Give the order of the layers which would be found if three layers of rock (1 2 3) were subjected to a fold, followed by a fault, and then the top layer was completely eroded. Assume that the sample goes deep enough to include all relevant layers. [2]

2, 3, (3), 2, 1, (1), 2, 3, (3), 2, 1 : allow repeated layers (shown in brackets)

Award 1 mark if the original ‘1’ is included.

(d) Which one of the four combinations described above could have caused it? You must also identify any layers which were omitted due to the sample being insufficiently deep. [2]

Fault then fold; (1 mark) bottom layer [1] omitted (1 mark).

1 mark for appreciation that it required a fault then a fold.

(e) Give two examples of samples with four layers which could not have been produced by the processes defined in this question. [2]

For example: 1, 4, 2, 4 and 1, 3, 1, 2 . Most easily achieved by a jump of two layers not

resulting from a fault (i.e. not highest to lowest)

1 mark for each impossible sample

(f) What can be concluded about the thicknesses of the three original layers which produced the following rock sample? [2]

Top layer = (greater than or equal to) 8 m

Middle layer = 9 m

Bottom layer = 2 m

2 out of 3 correct OR top layer eroded by (at least) 7m : 1 mark

SC [1] : total thickness is (greater than or equal to) 19m

Page 51: 9694 THINKING SKILLS - maxpapersmaxpapers.com/wp-content/uploads/2012/11/9694_w14_ms_complete… · 9694 THINKING SKILLS 9694/12 Paper 1 ... October/November 2014 9694 12 ... 1-mark

Page 3 Mark Scheme Syllabus Paper

Cambridge International A Level – October/November 2014 9694 33

© Cambridge International Examinations 2014

2 (a) (i) How did she distribute the overtime hours over the four weeks? [1]

She will do them all in the same week.

(ii) How much did her employer deduct at the end of the four weeks? [1]

30 × $4 = $120 basic per week, from which only $20 can be taken.

Total deduction = $25 + 3 × $20 = $85.

Allow follow through mark from (i).

(b) (i) If Hannah were now to work 5 hours overtime per four weeks, how much more would she get on pay day than if she did no overtime? [1]

Basic is $480 per month. The first $20 of overtime would be deducted, so working 5

hours at $5 per hour would only bring in $5 more.

(ii) What would Richard receive for the four-week period if Hannah did no overtime? [1]

4 × $20 = $80.

(c) (i) What is the minimum number of hours of overtime that Hannah would now have to work in order to get more on pay day than if she did no overtime? [1]

$480 – $410 = $70 is deducted from basic salary

$120 – $70 = $50

Therefore 11 hours.

Credit answers which clearly state that 10 hours of overtime would yield the same

amount on pay day.

(ii) What does Richard now receive on average each week? [1]

$70/4 = $17.50

(d) What percentage would be sufficient for him to receive $90 per four weeks if Hannah worked 4 hours of overtime each week? [2]

(4 × 30 × $4) + (4 × 4 × $5) = $480 + $80 = $560.

$560 – $410 = $150.

90/150 = 0.6 = 60% oe

1 mark for working with arithmetic error, or for concluding 90/560 = 16/17% with some

appropriate working.

Page 52: 9694 THINKING SKILLS - maxpapersmaxpapers.com/wp-content/uploads/2012/11/9694_w14_ms_complete… · 9694 THINKING SKILLS 9694/12 Paper 1 ... October/November 2014 9694 12 ... 1-mark

Page 4 Mark Scheme Syllabus Paper

Cambridge International A Level – October/November 2014 9694 33

© Cambridge International Examinations 2014

(e) Who benefits from this increase, and by how much? [2]

The increase is 4 × 30 × $0.2 = $24 per 4 weeks oe. Allow $94 as Richard’s new amount [1].

Depending on this result (or $504) seen, award 1 mark for all of this going to Richard.

SC – calculations with not more than one arithmetic error – allow follow through to 2nd

mark.

3 (a) Show that Samantha cannot avoid the tax collector if she travels without changing direction. [1]

Distance from the wall = 4 blocks. Distance around half the walls = 12 blocks.

(b) If Samantha decides to change direction once on the way out, (i) what is the greatest distance she could be from the tax collector when she reaches the wall? [1]

3 blocks

(ii) what is the probability that Samantha will avoid the tax collector? [2]

1/12: 1/6 chance she goes in the right direction initially, and 1/2 that she goes the right

way (left or right) after one block.

1 mark for 1/6 seen.

(c) List the points on the city wall where the tax collector could be if (i) he is not able to meet Tabitha, [2]

W, X, A, B, C

1 mark for either W and/or C missing, or D and/or V added.

(ii) he is able to meet Tabitha but not Endora. [2]

D, E, F

1 mark for explicitly comparing their sets of unattainable starting points – correctly

BCDEF and WXABC – and selecting an inappropriate subset OR for one extra point

added/omitted.

Page 53: 9694 THINKING SKILLS - maxpapersmaxpapers.com/wp-content/uploads/2012/11/9694_w14_ms_complete… · 9694 THINKING SKILLS 9694/12 Paper 1 ... October/November 2014 9694 12 ... 1-mark

Page 5 Mark Scheme Syllabus Paper

Cambridge International A Level – October/November 2014 9694 33

© Cambridge International Examinations 2014

(d) If the tax collector is waiting at point A, from how many of the 37 starting points within the city wall is it impossible for a citizen to avoid him, if the citizen travels without changing direction? [3]

There are 5 + 5 + 3 + 1 = 14 starting points from which it is impossible to avoid the tax

collector.

Award 1 mark for a clear attempt to divide the escape points up into those within 1, 2, 3 and

4 blocks from the walls, with at least one of these considered correctly. Award 2 marks if the

two of these cases are considered correctly.

Alternatively, award up to 2 marks for answers which analyse points which it is possible to

escape from (without changing direction) = 37 – 14 = 23 points. Award 1 mark if this is

considered systematically but with one arithmetic error.

SC: if the walls are included as starting points, then there are 15 starting points (2 marks)

(e) What is the greatest distance she could be from the tax collector when she reaches

the wall? Describe a possible route for her to take. [4]

9 blocks: In steps of 1 block at a time, towards M, then O, then X, then P, then W, then Q,

then L, then P.

Award 3 marks for a route taking 8 blocks: In steps of 1 block at a time, towards M, then S,

then N, then G, then M, then R, then N.

2 marks for 5 or 6 or 7 blocks with a matching route described.

1 mark for any route which allows her to escape the tax collector.

For any solutions which fit the descriptions above but for one erroneous step, deduct one

mark from the allotted mark (and continue to deduct for further erroneous steps).

4 (a) (i) How much will Matt charge Ferreb for the 44 parcels that he will pick up today? [2]

Answer: $275 (11 × $4 + 33 × $7)

If 2 marks cannot be awarded, award 1 mark for evidence of appreciation that 11 parcels

will be charged at the reduced rate / $4 and/or that 33 parcels will be charged at the full

rate / $7, OR (10@$4 and 34@$7 =) $206 OR (11@7 and 4@4 =) $93.

(ii) What is the maximum he could charge for picking up 44 parcels from one depot on any one day? [2]

Answer: $296 (40 × $7 + 4 × $4)

If 2 marks cannot be awarded, award 1 mark for evidence of appreciation that a

maximum of 40 parcels (40 × 10) can be charged at the full rate / $7.

Page 54: 9694 THINKING SKILLS - maxpapersmaxpapers.com/wp-content/uploads/2012/11/9694_w14_ms_complete… · 9694 THINKING SKILLS 9694/12 Paper 1 ... October/November 2014 9694 12 ... 1-mark

Page 6 Mark Scheme Syllabus Paper

Cambridge International A Level – October/November 2014 9694 33

© Cambridge International Examinations 2014

(b) How many parcels will Matt have on board when he leaves Payli? [2] Answer: 99 parcels

If 2 marks cannot be awarded, award 1 mark for sight of 44 (picked up) or 29 (dropped off),

or the calculation 14 + 8 + 11 + 10 + 12 + 17 + 8 + 5 + 14

(c) In what order must he visit the remaining islands, and how many parcels will he have

on board when he returns to Malzay? Justify your answer. [4]

Honia then Tolou (then Malzay), with a demonstration that the other route would entail

carrying more than 100 parcels - going to Tolou first would require him to carry 107 parcels.

[2 marks].

At Honia: drop off 42, pick up 40; 99 – 42 + 40 = 97

At Tolou: drop off 44, pick up 43; 97 – 44 + 43 = 96

2 marks for final correct answer; 1 mark for 97 given.

Condone 59, i.e. drops off 37 Malzay parcels.

(d) What is the total distance that Matt will fly today? [2]

Answer: 226 km (54 + 29 + 20 + 88 + 35) [follow through from incorrect (c)]

Tolou then Honia : 54 + 29 + 77 + 88 + 63 = 311 km

If 2 marks cannot be awarded, award 1 mark for sight of at least three correctly extracted

distances.

Page 55: 9694 THINKING SKILLS - maxpapersmaxpapers.com/wp-content/uploads/2012/11/9694_w14_ms_complete… · 9694 THINKING SKILLS 9694/12 Paper 1 ... October/November 2014 9694 12 ... 1-mark

Page 7 Mark Scheme Syllabus Paper

Cambridge International A Level – October/November 2014 9694 33

© Cambridge International Examinations 2014

(e) How many parcels in total did Matt pick up from Payli last Tuesday? [3] Answer: 48 (12 to each of the other four depots)

For every 4 parcels (one to each destination) he would have charged $26 ($5 to Honia, $7 to

Malzay, $5 to Styha and $9 to Tolou) under the old system.

For every 4 parcels (one to each destination) he now charges $28 (4 × $7) up to a total of 40,

then $16 (4 × $4).

12 × $26 = 10 × $28 + 4 × $4 = $296

If 3 marks cannot be awarded, award 2 marks for evidence of appreciation that 40 parcels

would have been $260 under the old system AND is $280 now,

OR an algebraic approach to the problem:

26p = (7 × 40) + 4(4p – 40) where p is the number dealt to each of the islands.

OR

Award 2 marks for two correct comparisons of costs for a specific number of parcels (ie

correct but incomplete trial and improvement).

If 2 marks cannot be awarded, award 1 mark for one correct comparison of costs for a

specific number of parcels (which could be simply $26 for 4 parcels under the old system and

$28 now).

If no other marks can be awarded, award 1 mark for an answer of 12.

Page 56: 9694 THINKING SKILLS - maxpapersmaxpapers.com/wp-content/uploads/2012/11/9694_w14_ms_complete… · 9694 THINKING SKILLS 9694/12 Paper 1 ... October/November 2014 9694 12 ... 1-mark

® IGCSE is the registered trademark of Cambridge International Examinations.

CAMBRIDGE INTERNATIONAL EXAMINATIONS

Cambridge International Advanced Level

MARK SCHEME for the October/November 2014 series

9694 THINKING SKILLS

9694/41 Paper 4 (Applied Reasoning), maximum raw mark 50

This mark scheme is published as an aid to teachers and candidates, to indicate the requirements of the examination. It shows the basis on which Examiners were instructed to award marks. It does not indicate the details of the discussions that took place at an Examiners’ meeting before marking began, which would have considered the acceptability of alternative answers. Mark schemes should be read in conjunction with the question paper and the Principal Examiner Report for Teachers. Cambridge will not enter into discussions about these mark schemes. Cambridge is publishing the mark schemes for the October/November 2014 series for most Cambridge IGCSE

®, Cambridge International A and AS Level components and some

Cambridge O Level components.

Page 57: 9694 THINKING SKILLS - maxpapersmaxpapers.com/wp-content/uploads/2012/11/9694_w14_ms_complete… · 9694 THINKING SKILLS 9694/12 Paper 1 ... October/November 2014 9694 12 ... 1-mark

Page 2 Mark Scheme Syllabus Paper

Cambridge International A Level – October/November 2014 9694 41

© Cambridge International Examinations 2014

1 Make five criticisms of the information provided by the food manufacturer and/or the inference that the snack is healthier than those of their competitors. [5]

Award 1 mark for any of the following:

• No information given about the proportion of fat in the snacks that is saturated.

• No information is given about the size of the snack so we do not know:

o how many would you eat at a time?

o the percentage of the snack’s mass that is fat.

• There is no indication of how much of a recommended daily intake of fat 1 g represents.

• Absence of comparison with fat content of other snack foods.

• The inference that they are “healthier” rests upon an assumption that the consumer would

otherwise be consuming a product that had a higher fat content.

• Without knowing what other ingredients there are in the product, we cannot make inferences

about it being healthier.

2 Briefly analyse Globe-trotter's argument in Document 1: Respect for Other Cultures, by identifying its main conclusion and main reasons, as well as any intermediate conclusions and counter-arguments. [6]

MR/IC – those who feel prejudice against others are likely to commit hate crimes.

IC – all long-surviving cultures are good cultures.

IC – Attempting to modify other people’s cultures carelessly means destabilising stable structures

that you don’t understand and may not be able to replace with anything near equal.

IC – As a host country we should not expect people who come here from other countries to

change their cultures and assimilate.

IC – This calls for a great deal of broadmindedness in societies that have been traditionally more

or less mono-cultural.

CA – It is only natural that people of host countries view the incoming cultures as intrusions.

and/or when a mono-cultural society is bombarded with immigrants, various prejudices,

misunderstandings and xenophobic attitudes come to the fore, contributing to tensions.

IC – The way forward is to replace this ignorance with understanding of others.

CA – We cannot be expected to understand all cultures completely – nor is there any rule stating

that we should.

IC – A multicultural society can exist peacefully only when there is acceptance and tolerance of

all those cultural practices that defy understanding.

MC – So we need to tolerate culturally different neighbours and their oddities.

Marks 1 mark for each element (maximum 3 if MC not identified).

Page 58: 9694 THINKING SKILLS - maxpapersmaxpapers.com/wp-content/uploads/2012/11/9694_w14_ms_complete… · 9694 THINKING SKILLS 9694/12 Paper 1 ... October/November 2014 9694 12 ... 1-mark

Page 3 Mark Scheme Syllabus Paper

Cambridge International A Level – October/November 2014 9694 41

© Cambridge International Examinations 2014

3 Give a critical evaluation of Globe-trotter's argument in Document 1: Respect for Other

Cultures, by identifying and explaining any flaws, implicit assumptions and other weaknesses. [9]

Para 1 Stipulative definition – “Prejudice is really hatred of humanity”

Slippery slope from feeling prejudice to being likely to commit hate crimes

Contradiction – stating that cultures are stable contradicts previous statement that cultures are

constantly evolving.

The second half of the paragraph includes much vague and circular reasoning.

Para 2 Conflation of those who “abandon their cultures” with those who “change their cultures”.

Assumption that people are only able to ‘fit in’ with those from their native culture.

Slippery slope from adopting a different culture to becoming an aimless individual.

Para 4 Conflation of “ignorance” with “prejudice”/feeling “scared” or “undermined”.

The meaning of the terms used in the paragraph is rather confused and shifting; in particular,

lacking prejudice seems to be equated with being civilised.

For the last statement to be true, it would have to be assumed that “understanding of others”

entails the elimination of all feelings of being scared or undermined.

Para 5 Contradiction – paragraph 4 states that one should understand another culture, whereas here

this is described as an impossible task.

Overall Evaluation

Overall, the argument is mediocre. Although most of the intermediate conclusions do support the

main conclusion, there are flaws in the reasoning supporting these and the MC is not at all

persuasive for someone who does not agree with many of the propositions – particularly because

of the contradiction between paragraphs 4 and 5. The counter position has not been represented

sufficiently or refuted convincingly.

Marks For each sound evaluative point 1 mark and 2 marks for a developed point, to a maximum of 8

marks.

Up to 2 marks for an overall judgment on the argument.

(Maximum 9 marks)

Page 59: 9694 THINKING SKILLS - maxpapersmaxpapers.com/wp-content/uploads/2012/11/9694_w14_ms_complete… · 9694 THINKING SKILLS 9694/12 Paper 1 ... October/November 2014 9694 12 ... 1-mark

Page 4 Mark Scheme Syllabus Paper

Cambridge International A Level – October/November 2014 9694 41

© Cambridge International Examinations 2014

4 ‘We should fight prejudice by making it a crime.’ To what extent do you agree with this statement? Construct a well-reasoned argument in support of your view, commenting critically

on some or all of Documents 1 to 5, and introducing ideas of your own. [30]

Level Structure Max

8 Quality of argument

Max 8

Use of documents Max

8 Treatment of counter

positions Max

6

4 Precise conclusion and accomplished argument structure with consistent use of intermediate conclusions. Likely to include at least two of the following:

• strands of reasoning

• suppositional reasoning

• analogy

• evidence

• examples Argument is structured so the thought process is made clear. Uses vocabulary of reasoning appropriately and effectively to support argument.

7–8 Cogent and convincing reasoning which answers the question which was asked. Subtle thinking about the issue. Use of relevant own ideas and ideas from documents. Very few significant gaps or flaws.

7–8 Perceptive, relevant and accurate use of documents to support reasoning. Sustained and confident evaluation of documents to support reasoning. (Two or more valid evaluative references to documents). Able to combine information from two or more documents and draw a precise inference.

7–8 Consideration of key counter arguments and effective response to these. Use of own ideas in response to counter arguments. Use of valid critical tools to respond to counter arguments. Effective use of appropriate terminology.

5–6

3 Clear conclusion that is more than “I agree”. Clear argument structure, which may be simple and precise or attempt complexity with some success. Appropriate use of intermediate conclusions. Use of other argument elements to support reasoning. Generally makes thinking clear. Appropriate use of vocabulary of reasoning.

5–6 Effective and persuasive reasoning which answers the question which was asked. (Although there may be some irrelevance or reliance on dubious assumptions.) Use of own ideas and ideas from documents. Few significant gaps or flaws.

5–6 Relevant and accurate use of documents which supports reasoning. (Must reference 3+ documents.) Some evaluation and comparison of documents to support reasoning. Inference drawn from ≥ 1 document.

5–6 Consideration of key counter arguments and effective response to these. Some use of appropriate terminology.

3–4

Page 60: 9694 THINKING SKILLS - maxpapersmaxpapers.com/wp-content/uploads/2012/11/9694_w14_ms_complete… · 9694 THINKING SKILLS 9694/12 Paper 1 ... October/November 2014 9694 12 ... 1-mark

Page 5 Mark Scheme Syllabus Paper

Cambridge International A Level – October/November 2014 9694 41

© Cambridge International Examinations 2014

2 Conclusion stated but may be “I agree”. Sufficient clarity for meaning to be clear throughout. Structure may be easy to follow but brief or a longer argument which has a less clear structure. Uses reasons. Some appropriate use of vocabulary of reasoning.

3–4 A reasoned stance which attempts to answer the question which was asked. Some support for the conclusion. (Although there may be considerable irrelevance or reliance on dubious assumptions.) Some thinking/own ideas about the issue. Use of rhetorical questions and emotive language. Some significant gaps or flaws.

3–4 Some relevant use of documents to support reasoning, but some documents used indiscriminately. Some (perhaps implicit) comparison of documents or some critical evaluation of documents.

3–4 Inclusion of counter argument or counter assertion but response to this is ineffective.

2

1 Attempt to construct an argument. Unclear conclusion, multiple conclusions or no conclusion. Disjointed, incoherent reasoning. Use of examples in place of reasoning. Possibly a discourse or a rant. Reasons presented with no logical connection. Documents considered sequentially. Substantial irrelevant material.

1–2 Attempt to answer the general thrust of the question. Attempt to support their view. Excessive use of rhetorical questions and emotive language. Ideas which are contradictory.

1–2 Some use, perhaps implicit, use of documents. No attempt at critical evaluation. No comparison of documents.

1–2 Inclusion of counter argument or counter assertion with no response.

1

Page 61: 9694 THINKING SKILLS - maxpapersmaxpapers.com/wp-content/uploads/2012/11/9694_w14_ms_complete… · 9694 THINKING SKILLS 9694/12 Paper 1 ... October/November 2014 9694 12 ... 1-mark

® IGCSE is the registered trademark of Cambridge International Examinations.

CAMBRIDGE INTERNATIONAL EXAMINATIONS

Cambridge International Advanced Level

MARK SCHEME for the October/November 2014 series

9694 THINKING SKILLS

9694/42 Paper 4 (Applied Reasoning), maximum raw mark 50

This mark scheme is published as an aid to teachers and candidates, to indicate the requirements of the examination. It shows the basis on which Examiners were instructed to award marks. It does not indicate the details of the discussions that took place at an Examiners’ meeting before marking began, which would have considered the acceptability of alternative answers. Mark schemes should be read in conjunction with the question paper and the Principal Examiner Report for Teachers. Cambridge will not enter into discussions about these mark schemes. Cambridge is publishing the mark schemes for the October/November 2014 series for most Cambridge IGCSE

®, Cambridge International A and AS Level components and some

Cambridge O Level components.

Page 62: 9694 THINKING SKILLS - maxpapersmaxpapers.com/wp-content/uploads/2012/11/9694_w14_ms_complete… · 9694 THINKING SKILLS 9694/12 Paper 1 ... October/November 2014 9694 12 ... 1-mark

Page 2 Mark Scheme Syllabus Paper

Cambridge International A Level – October/November 2014 9694 42

© Cambridge International Examinations 2014

1 Make five criticisms of the information provided by the food manufacturer and/or the inference that the snack is healthier than those of their competitors. [5]

Award 1 mark for any of the following:

• No information given about the proportion of fat in the snacks that is saturated.

• No information is given about the size of the snack so we do not know: o how many would you eat at a time?

o the percentage of the snack’s mass that is fat.

• There is no indication of how much of a recommended daily intake of fat 1 g represents.

• Absence of comparison with fat content of other snack foods.

• The inference that they are “healthier” rests upon an assumption that the consumer would otherwise be consuming a product that had a higher fat content.

• Without knowing what other ingredients there are in the product, we cannot make inferences about it being healthier.

2 Briefly analyse Globe-trotter's argument in Document 1: Respect for Other Cultures, by

identifying its main conclusion and main reasons, as well as any intermediate conclusions and counter-arguments. [6]

MR/IC – those who feel prejudice against others are likely to commit hate crimes. IC – all long-surviving cultures are good cultures. IC – Attempting to modify other people’s cultures carelessly means destabilising stable structures

that you don’t understand and may not be able to replace with anything near equal. IC – As a host country we should not expect people who come here from other countries to

change their cultures and assimilate. IC – This calls for a great deal of broadmindedness in societies that have been traditionally more

or less mono-cultural. CA – It is only natural that people of host countries view the incoming cultures as intrusions.

and/or when a mono-cultural society is bombarded with immigrants, various prejudices, misunderstandings and xenophobic attitudes come to the fore, contributing to tensions.

IC – The way forward is to replace this ignorance with understanding of others. CA – We cannot be expected to understand all cultures completely – nor is there any rule stating

that we should. IC – A multicultural society can exist peacefully only when there is acceptance and tolerance of

all those cultural practices that defy understanding. MC – So we need to tolerate culturally different neighbours and their oddities. Marks 1 mark for each element (maximum 3 if MC not identified). Where ellipsis is used assume all the words within the section have been written out. Where an IC is identified as a (main) reason, do not credit the first instance of this but thereafter

credit ICs identified as (main) reasons. Credit MR identified as a reason/premise. Do not credit MR identified as an IC. Do not credit any

unidentified argument elements. Accept close paraphrase with identical meaning.

Page 63: 9694 THINKING SKILLS - maxpapersmaxpapers.com/wp-content/uploads/2012/11/9694_w14_ms_complete… · 9694 THINKING SKILLS 9694/12 Paper 1 ... October/November 2014 9694 12 ... 1-mark

Page 3 Mark Scheme Syllabus Paper

Cambridge International A Level – October/November 2014 9694 42

© Cambridge International Examinations 2014

3 Give a critical evaluation of Globe-trotter's argument in Document 1: Respect for Other Cultures, by identifying and explaining any flaws, implicit assumptions and other weaknesses. [9]

Para 1 Stipulative definition – “Prejudice is really hatred of humanity” Slippery slope from feeling prejudice to being likely to commit hate crimes Contradiction – stating that cultures are stable contradicts previous statement that cultures are

constantly evolving. The second half of the paragraph includes much vague and circular reasoning. Para 2 Conflation of those who “abandon their cultures” with those who “change their cultures”. Assumption that people are only able to ‘fit in’ with those from their native culture. Slippery slope from adopting a different culture to becoming an aimless individual. Para 4 Conflation of “ignorance” with “prejudice”/feeling “scared” or “undermined”. The meaning of the terms used in the paragraph is rather confused and shifting; in particular,

lacking prejudice seems to be equated with being civilised. For the last statement to be true, it would have to be assumed that “understanding of others”

entails the elimination of all feelings of being scared or undermined. Para 5 Contradiction – paragraph 4 states that one should understand another culture, whereas here

this is described as an impossible task. Overall Evaluation Overall, the argument is mediocre. Although most of the intermediate conclusions do support the

main conclusion, there are flaws in the reasoning supporting these and the MC is not at all persuasive for someone who does not agree with many of the propositions – particularly because of the contradiction between paragraphs 4 and 5. The counter position has not been represented sufficiently or refuted convincingly.

Marks For each sound evaluative point 1 mark and 2 marks for a developed point, to a maximum of 8

marks. Up to 2 marks for an overall judgment on the argument. (Maximum 9 marks)

Page 64: 9694 THINKING SKILLS - maxpapersmaxpapers.com/wp-content/uploads/2012/11/9694_w14_ms_complete… · 9694 THINKING SKILLS 9694/12 Paper 1 ... October/November 2014 9694 12 ... 1-mark

Page 4 Mark Scheme Syllabus Paper

Cambridge International A Level – October/November 2014 9694 42

© Cambridge International Examinations 2014

4 ‘We should fight prejudice by making it a crime.’ To what extent do you agree with this statement? Construct a well-reasoned argument in support of your view, commenting critically

on some or all of Documents 1 to 5, and introducing ideas of your own. [30]

Level Structure Max

8 Quality of argument

Max 8

Use of documents Max

8 Treatment of counter

positions Max

6

4 Precise conclusion and accomplished argument structure with consistent use of intermediate conclusions. Likely to include at least two of the following:

• strands of reasoning

• suppositional reasoning

• analogy

• evidence

• examples Argument is structured so the thought process is made clear. Uses vocabulary of reasoning appropriately and effectively to support argument.

7–8 Cogent and convincing reasoning which answers the question which was asked. Subtle thinking about the issue. Use of relevant own ideas and ideas from documents. Very few significant gaps or flaws.

7–8 Perceptive, relevant and accurate use of documents to support reasoning. Sustained and confident evaluation of documents to support reasoning. (Two or more valid evaluative references to documents). Able to combine information from two or more documents and draw a precise inference.

7–8 Consideration of key counter arguments and effective response to these. Use of own ideas in response to counter arguments. Use of valid critical tools to respond to counter arguments. Effective use of appropriate terminology.

5–6

3 Clear conclusion that is more than “I agree”. Clear argument structure, which may be simple and precise or attempt complexity with some success. Appropriate use of intermediate conclusions. Use of other argument elements to support reasoning. Generally makes thinking clear. Appropriate use of vocabulary of reasoning.

5–6 Effective and persuasive reasoning which answers the question which was asked. (Although there may be some irrelevance or reliance on dubious assumptions.) Use of own ideas and ideas from documents. Few significant gaps or flaws.

5–6 Relevant and accurate use of documents which supports reasoning. (Must reference 3+ documents.) Some evaluation and comparison of documents to support reasoning. Inference drawn from ≥ 1 document.

5–6 Consideration of key counter arguments and effective response to these. Some use of appropriate terminology.

3–4

Page 65: 9694 THINKING SKILLS - maxpapersmaxpapers.com/wp-content/uploads/2012/11/9694_w14_ms_complete… · 9694 THINKING SKILLS 9694/12 Paper 1 ... October/November 2014 9694 12 ... 1-mark

Page 5 Mark Scheme Syllabus Paper

Cambridge International A Level – October/November 2014 9694 42

© Cambridge International Examinations 2014

2 Conclusion stated but may be “I agree”. Sufficient clarity for meaning to be clear throughout. Structure may be easy to follow but brief or a longer argument which has a less clear structure. Uses reasons. Some appropriate use of vocabulary of reasoning.

3–4 A reasoned stance which attempts to answer the question which was asked. Some support for the conclusion. (Although there may be considerable irrelevance or reliance on dubious assumptions.) Some thinking/own ideas about the issue. Use of rhetorical questions and emotive language. Some significant gaps or flaws.

3–4 Some relevant use of documents to support reasoning, but some documents used indiscriminately. Some (perhaps implicit) comparison of documents or some critical evaluation of documents.

3–4 Inclusion of counter argument or counter assertion but response to this is ineffective.

2

1 Attempt to construct an argument. Unclear conclusion, multiple conclusions or no conclusion. Disjointed, incoherent reasoning. Use of examples in place of reasoning. Possibly a discourse or a rant. Reasons presented with no logical connection. Documents considered sequentially. Substantial irrelevant material.

1–2 Attempt to answer the general thrust of the question. Attempt to support their view. Excessive use of rhetorical questions and emotive language. Ideas which are contradictory.

1–2 Some use, perhaps implicit, use of documents. No attempt at critical evaluation. No comparison of documents.

1–2 Inclusion of counter argument or counter assertion with no response.

1

Page 66: 9694 THINKING SKILLS - maxpapersmaxpapers.com/wp-content/uploads/2012/11/9694_w14_ms_complete… · 9694 THINKING SKILLS 9694/12 Paper 1 ... October/November 2014 9694 12 ... 1-mark

® IGCSE is the registered trademark of Cambridge International Examinations.

CAMBRIDGE INTERNATIONAL EXAMINATIONS

Cambridge International Advanced Level

MARK SCHEME for the October/November 2014 series

9694 THINKING SKILLS

9694/43 Paper 4 (Applied Reasoning), maximum raw mark 50

This mark scheme is published as an aid to teachers and candidates, to indicate the requirements of the examination. It shows the basis on which Examiners were instructed to award marks. It does not indicate the details of the discussions that took place at an Examiners’ meeting before marking began, which would have considered the acceptability of alternative answers. Mark schemes should be read in conjunction with the question paper and the Principal Examiner Report for Teachers. Cambridge will not enter into discussions about these mark schemes. Cambridge is publishing the mark schemes for the October/November 2014 series for most Cambridge IGCSE

®, Cambridge International A and AS Level components and some

Cambridge O Level components.

Page 67: 9694 THINKING SKILLS - maxpapersmaxpapers.com/wp-content/uploads/2012/11/9694_w14_ms_complete… · 9694 THINKING SKILLS 9694/12 Paper 1 ... October/November 2014 9694 12 ... 1-mark

Page 2 Mark Scheme Syllabus Paper

Cambridge International A Level – October/November 2014 9694 43

© Cambridge International Examinations 2014

1 Make five criticisms of either the statistics in the passage and/or the inference drawn from them. [5]

Award 1 mark for any of the following:

• The use of numbers of deaths is meaningless if we do not know the number of competitors, percentages would be more useful.

• The number of deaths might not be significantly different from the number of deaths in a similarly sized population over a similar time.

• Only two examples given from two events, it is likely that more races have taken place over the time period suggested.

• There should be a much better way of measuring the injuriousness of fun runs than just picking examples – some kind of average per competitor per mile would be more useful.

• It is likely that the two examples quoted have been selected as ‘worst cases’.

• Without knowing about the health of the runners who died we cannot judge whether their deaths were a direct result of the run.

• “As much as 26 miles” – is meaningless as many events could be over a much shorter distance.

• The two quoted events are both over 13 miles and do not, therefore, support any inference about the majority of fun runs which are shorter events.

• Two is a very low number of reports on which to base a general conclusion.

• It could be that all the danger is due to the running, and the mass-participation element is irrelevant.

• Since any activity is “potentially dangerous”, the inference is rather vacuous.

2 Briefly analyse Bank-lover’s argument in Document 1: Stop Bashing Banks, by identifying its main conclusion and main reasons, as well as any intermediate conclusions and counter-arguments. [6]

IC – Nations can have no future without banks.

IC – Gifted young entrepreneurs need the financial backing of banks to launch their innovations.

IC – (It is also the reason why,) in order for countries to compete, they need a confident and dynamic banking sector.

IC – Banks help improve the quality of life all round.

IC – people should not turn their aggression on the banks themselves. MR – the banking sector is utterly crucial to a nation’s economy and the current system of global

capitalism.

CA – we should impose stringent regulation on banks and keep them under strict surveillance. MC – It is (, then,) very important that we move towards deregulating our banks.

Marks 1 mark for each element (maximum 4 if MC not identified).

Where ellipsis is used assume all the words within the section have been written out.

Where an IC is identified as a (main) reason, do not credit the first instance of this but thereafter credit ICs identified as (main) reasons.

Credit MR identified as a reason / premise. Do not credit MR identified as an IC. Do not credit any unidentified argument elements.

Page 68: 9694 THINKING SKILLS - maxpapersmaxpapers.com/wp-content/uploads/2012/11/9694_w14_ms_complete… · 9694 THINKING SKILLS 9694/12 Paper 1 ... October/November 2014 9694 12 ... 1-mark

Page 3 Mark Scheme Syllabus Paper

Cambridge International A Level – October/November 2014 9694 43

© Cambridge International Examinations 2014

3 Give a critical evaluation of the strength of Bank-lover’s argument in Document 1: Stop Bashing Banks, by identifying and explaining any flaws, implicit assumptions and other weaknesses. [9]

Para 1 Conflation of reckless gambling by banks with all banking activities. Slippery slope from banks withdrawing finance to having no future. To say that “the jobs and investments generated by the banking sector are simply irreplaceable”

is hyperbole. Para 2 The author treats “the right idea and financial backing” as sufficient for success, although this is

by no means clear and yet his argument rests heavily on this point. The argument shifts from the requirement of financial backing to the financial backing of banks;

there may well be other sources of financial backing. Para 3 The example given is weak, as films are unlikely to reflect real life. Assumption that the “bank guy” mentioned perceived the loan he made as taking a risk. It is not totally clear that a “confident and dynamic” banking sector will necessarily be a risk-taking

one. Assumption that the only way for countries to compete is to innovate/encourage start-ups. Para 4 Even if software applications can offer some benefits, it is not the case that they (and therefore

banks) can improve “the quality of life all round”. Para 5 ad hominem – “unthinking opponents”. Inconsistency – arguing in favour of accountability (“put their necks on the line”) conflicts with

the conclusion in favour of deregulation. Straw man – the author is misrepresenting the concerns of the bank-bashers as being limited to

outrage over those committing serious fraud. Inconsistency – arguing in favour of sanctions for fraudsters conflicts with the conclusion in

favour of deregulation. Red herring – The author suggests that people have turned their aggression on the banks

because of the activities of fraudsters, but makes an irrelevant reply that banks are utterly crucial to the economy.

Page 69: 9694 THINKING SKILLS - maxpapersmaxpapers.com/wp-content/uploads/2012/11/9694_w14_ms_complete… · 9694 THINKING SKILLS 9694/12 Paper 1 ... October/November 2014 9694 12 ... 1-mark

Page 4 Mark Scheme Syllabus Paper

Cambridge International A Level – October/November 2014 9694 43

© Cambridge International Examinations 2014

Para 6 ad hominem – “short-sighted fools”. The author suggests that regulating banks will make them “too nervous to lend”, whereas it

seems likely that regulation may engender a requirement to lend. The argument here is against increased regulation, and offers no support for the notion that there

should be less regulation. Much of the reasoning argues that a vigorous banking sector is essential without explaining the

role that deregulation would have in support of this. Overall evaluation The argument is overall weak, one-sided and evasive, failing to demonstrate that deregulation will

achieve the stated benefits to the economy and society. It fails to look at the reasons behind the public outcry against banks, and also fails to consider any of the negative consequences of increased risk-taking. The author confuses reckless gambling risk with calculated risk.

Marks For each sound evaluative point 1 mark and 2 marks for a developed point, to a maximum of 8

marks. Up to 2 marks for an overall judgment on the argument. (Maximum 9 marks)

Page 70: 9694 THINKING SKILLS - maxpapersmaxpapers.com/wp-content/uploads/2012/11/9694_w14_ms_complete… · 9694 THINKING SKILLS 9694/12 Paper 1 ... October/November 2014 9694 12 ... 1-mark

Page 5 Mark Scheme Syllabus Paper

Cambridge International A Level – October/November 2014 9694 43

© Cambridge International Examinations 2014

4 ‘We need to radically change our banking system.’ To what extent do you agree with this statement? Construct a well-reasoned argument in support of your view, commenting critically

on some or all of Documents 1 to 5, and introducing ideas of your own. [30]

Level Structure Max

8 Quality of argument

Max 8

Use of documents Max

8 Treatment of counter

positions Max

6

4 Precise conclusion and accomplished argument structure with consistent use of intermediate conclusions. Likely to include at least two of the following:

• strands of reasoning

• suppositional reasoning

• analogy

• evidence

• examples Argument is structured so the thought process is made clear. Uses vocabulary of reasoning appropriately and effectively to support argument.

7–8 Cogent and convincing reasoning which answers the question which was asked. Subtle thinking about the issue. Use of relevant own ideas and ideas from documents. Very few significant gaps or flaws.

7–8 Perceptive, relevant and accurate use of documents to support reasoning. Sustained and confident evaluation of documents to support reasoning. (Two or more valid evaluative references to documents). Able to combine information from two or more documents and draw a precise inference.

7–8 Consideration of key counter arguments and effective response to these. Use of own ideas in response to counter arguments. Use of valid critical tools to respond to counter arguments. Effective use of appropriate terminology.

5–6

3 Clear conclusion that is more than “I agree”. Clear argument structure, which may be simple and precise or attempt complexity with some success. Appropriate use of intermediate conclusions. Use of other argument elements to support reasoning. Generally makes thinking clear. Appropriate use of vocabulary of reasoning.

5–6 Effective and persuasive reasoning which answers the question which was asked. (Although there may be some irrelevance or reliance on dubious assumptions.) Use of own ideas and ideas from documents. Few significant gaps or flaws.

5–6 Relevant and accurate use of documents which supports reasoning. (Must reference 3+ documents.) Some evaluation and comparison of documents to support reasoning. Inference drawn from ≥ 1 document.

5–6 Consideration of key counter arguments and effective response to these. Some use of appropriate terminology.

3–4

Page 71: 9694 THINKING SKILLS - maxpapersmaxpapers.com/wp-content/uploads/2012/11/9694_w14_ms_complete… · 9694 THINKING SKILLS 9694/12 Paper 1 ... October/November 2014 9694 12 ... 1-mark

Page 6 Mark Scheme Syllabus Paper

Cambridge International A Level – October/November 2014 9694 43

© Cambridge International Examinations 2014

Level Structure Max

8 Quality of argument

Max 8

Use of documents Max

8 Treatment of counter

positions Max

6

2 Conclusion stated but may be “I agree”. Sufficient clarity for meaning to be clear throughout. Structure may be easy to follow but brief or a longer argument which has a less clear structure. Uses reasons. Some appropriate use of vocabulary of reasoning.

3–4 A reasoned stance which attempts to answer the question which was asked. Some support for the conclusion. (Although there may be considerable irrelevance or reliance on dubious assumptions.) Some thinking/own ideas about the issue. Use of rhetorical questions and emotive language. Some significant gaps or flaws.

3–4 Some relevant use of documents to support reasoning, but some documents used indiscriminately. Some (perhaps implicit) comparison of documents or some critical evaluation of documents.

3–4 Inclusion of counter argument or counter assertion but response to this is ineffective.

2

1 Attempt to construct and argument. Unclear conclusion, multiple conclusions or no conclusion. Disjointed, incoherent reasoning. Use of examples in place of reasoning. Possibly a discourse or a rant. Reasons presented with no logical connection. Documents considered sequentially. Substantial irrelevant material.

1–2 Attempt to answer the general thrust of the question. Attempt to support their view. Excessive use of rhetorical questions and emotive language. Ideas which are contradictory.

1–2 Some use, perhaps implicit, use of documents. No attempt at critical evaluation. No comparison of documents.

1–2 Inclusion of counter argument or counter assertion with no response

1